You are on page 1of 75

neet 2024 SPeciAL

CHAPTE R
LEVEL - I
03
Human Reproduction

Content Writer 7905239992


This type of material is available with your name brand, contact me EDU+ 7905239992

Biology

CONTENTS
MALE REPRODUCTIVE SYSTEM

FEMALE REPRODUCTIVE SYSTEM


HUMAN REPRODUCTION

GAMETOGENESIS

MENSTRUAL CYCLE

FERTILISation AND IMPLANTATION

PREGNANCY AND EMBRYONIC


DEVELOPMENT

PARTURITION AND LACTATION

“An original is a creation by desire. Any reproduction of an originals


motivated be necessity. It is marvelous that we are the only species
that creates gratuitous forms. To create is divine, to reproduce is
human.”

—Man Ray

Human Reproduction

3.1 MALE REPRODUCTIVE SYSTEM


‹ It is located in the pelvis region. It has the following structures:

Male Reproductive System

A pair of
Accessory Accessory External
testes
ducts glands genitalia
sing. testis

Testes are located outside the abdomen


in a pouch called the scrotum. Testes
are oval in shape. 4-5 cm in length and
Bladder 2-3 cm in width.(AIPMT 2011)
Pubic symphysis
Testicular lobules are the
Vas deferens compartments present in the each testis
(250 compartments), each containing 1-3
Urethra
highly coiled tubules called seminiferous
tubules.
Corpus
spongiosum
Anus Scrotum helps in maintaining 2-2.5°C
Bulbourethral
gland Corpus
lower temperature than the internal
Pelvic floor
muscle cavernosum body to produce sperms.

Seminal Epididymis Through the inguinal canal, scrotum


vesicle Urethral remains attached to the abdominal
Prostate Testicle opening cavity.

Fig. 3.1.1 Male reproductive system

Vas (ductus)
deferens Straight tubule (tubuli recti)
Testis covered
Externally
with 3 Head of
by tunica
protective epididymis
vaginalis Seminiferous tubule
coverings
Vasa
efferentia Cavity

Rete testis Visceral layer Tunica


In the middle vaginalis
by the tunica Internally Body of
Parietal layer
albuginea by tunica epididymis
(white fibrous vasculosa Tunica albuginea
membrane) Tail of
epididymis
Fig. 3.1.2 Section of testis

01
Biology

‹ The seminiferous tubule is lined by two cells, which are spermatogonia and Sertoli cells.
seminiferous tubules

Undergoes meiosis to form


Male germ cells or spermatogonia
spermatozoa (sperm cells)
Cells of

Sertoli cells (AIPMT 2010) Provides nutrition to germ cells

‹ Leydig cells or interstitial cells and blood vessels are present in the region outside the
seminiferous tubules (interstitial spaces). Leydig cells synthesise and secrete androgens
(testosterone). (AIPMT 2012)

1
Sperms are carried by a network of
tubules (rete testis) from seminiferous
tubules to vasa efferentia. (NEET 2019)

2
Vasa efferentia conducts sperms to the
epididymis (site of sperm maturation and
storage till ejaculation). (AIPMT 2011)
Rete testis

Vasa
Urethra efferentia

3
Vasa deferentia (sing. vas deferens)
arise from epididymis which ascends Accessory
to the abdomen and loops over urinary ducts are
bladder.
Ejaculatory
ducts Epididymis

Vasa

4
Vas deferens unites with the duct deferentia
coming from the seminal vesicle thus
forming an ejaculatory duct, which
opens into urethra.

Urethra originates from the urinary bladder

5 and opens to extend through the penis to


the external opening called the urethral
meatus.

02
Human Reproduction

‹ Male accessory glands are:

• One pair of sac-like structure present near the base of


bladder.
• Produce an alkaline secretion (pH 7.4)
Seminal


vesicles • Alkaline nature neutralises the acidity of the female


reproductive tract.
• Secretion contains fructose, clotting proteins (different


those in the blood), and prostaglandin hormone.

Single large gland


Male • Produces milky secretion with pH 6.5, which contains citric
Prostate

accessory acid, acid phosphatase, amylase, and pepsinogen.
gland
glands • Its secretion nourishes and helps in the transport of

sperm.

• Present in a pair on either side of the membranous


Cowper’s or

urethra.
bulbourethral
glands • Secretes mucus that lubricates the end of penis and

the urethral lining.

Secretions of these glands constitute the seminal plasma which is NCERT


Highlights
rich in fructose, calcium and certain enzymes. (AIPMT 2012)

‹ The male external genitalia and main copulatory organ is the penis. It transfers the sperm
cells during sexual intercourse.

‹ The tip of the penis is called the glans penis, which is covered by a loose fold of skin, the
foreskin.

Beyond NCERT
The erectile tissues of the penis are three: Two dorsal corpus
cavernosa and one ventral corpus spongiosum. They are
surrounded by fibrous tissue. The penis carries both urine and
sperms.

03
Biology

Vas
deferens

FANTA FACT
Fructose which is present in semen


is used for forensic tests for rape.
The presence of it inside the female
reproductive tract confirms sexual
intercourse.
Fig. 3.1.3 Front view of male reproductive system (AIPMT 2009)

3.2 FEMALE REPRODUCTIVE SYSTEM


‹ The female reproductive system has:

Female Reproductive System

Accessory External Accessory


Ovaries
ducts genitalia glands

Ovaries are the primary female sex


organs and occur in pairs located
Uterine fundus on either side of the abdomen. It
Uterine cavity
Isthmus produces female gamete (ovum) and
Fallopian
ovarian hormones.
Ampulla
tube
Infundibulum
Each ovary is 2-4 cm in length and is
Endometrium Ovary
Myometrium connected to the pelvic wall and the
Fimbriae
Perimetrium uterus by ligaments.
Cervix
Cervical canal
Ovary is covered by a thin epithelium
Vagina
which encloses the stroma of the ovary.
Fig. 3.2.1 Female reproductive Stroma is divided into a peripheral
system cortex and an inner medulla.

04
Human Reproduction

‹ Fallopian tubes are 10-12 cm long, ciliated, muscular,


Fallopian
and tubular structures, that conducts ovum to the tubes
uterus. (oviducts)

‹ It has a funnel-shaped structure called the infundibulum


that lies near the ovary and has finger-like projections
Uterus
called fimbriae, which collects ovum during ovulation.
(AIPMT 2010) Accessory
ducts are

‹ Ampulla is the wider part located after the infundibulum,
and isthmus is the last, narrow part of the oviduct which Vagina

opens into the uterus.

NCERT
Highlights Ampullary region is the site of Cervix
fertilisation.

It is divided into Uterus opens into


the vagina through Vagina is about
Uterus is shaped fundus
cervix. The cavity 7.5 cm long,
like an inverted (upper dome
of the cervix is the and it facilitates
pear and is also shaped), body or
cervical canal, menstrual flow and
known as the cavity (middle part)
which along with acts as copulation
womb. and cervix (narrow the vagina forms canal during coitus.
and long part). the birth canal.

Uterus wall has three


layers of tissue

Myometrium Endometrium
Perimetrium (outer
(middle layer of (inner glandular
membranous layer)
smooth muscles) layer)

• It exhibits contraction during • It lines




parturition the uterine
cavity and
undergoes
Beyond NCERT cyclic
changes
The surgical removal of the during
uterus is called hysterectomy. menstrual
cycle

05
Biology

3.2.1 EXTERNAL GENITALIA


‹ External genitalia are collectively called vulva and have the following structures.

External genitalia

Mons pubis Labia majora Labia minora

• Fleshy folds of

skin which extends
• Cushion of fatty
from mons pubis • These are paired

tissue


surrounding the folds of tissue under
• Covered by skin and
vaginal orifice the labia majora

pubic hair
• Homologous to

scrotum

Clitoris
‹ The opening of the vagina is often
covered partially by a membrane called
Labia minora Urethral opening
the hymen.
Labia majora
Vagina
‹ A tiny finger-like structure, which lies
at the upper junction of the two labia
minora above the urethral opening is
called the clitoris.
Anus
Fig. 3.2.2 External genitalia

Beyond NCERT
Vestibular glands are of two types:
1. Skene glands are numerous minute glands present on either


side of the urethral orifice.


2. Bartholin’s glands are paired glands, located on each side of


vaginal orifice.
These glands are homologous to Cowper’s glands in males, which
provides lubrication during coitus.

06
Human Reproduction

3.2.2 MAMMARY GLANDS


‹ These are paired structures (breasts) that contain glandular tissues and a variable amount of fat.
‹ The areola is the pigmented circular area of the skin surrounding the nipple.
‹ The glandular tissue of each breast is divided into 15-20 mammary lobes containing
clusters of cells called the alveoli; cells of the alveoli secrete milk, which is stored in the
cavities (lumens) of alveoli.
‹ The alveoli open into the mammary tubules. The tubules of each lobe combine to form a
mammary duct.
‹ Several mammary ducts join to form a wider mammary ampulla which is connected to the
lactiferous duct through which milk is sucked out.

Beyond NCERT
The main constituents of human milk are fat droplets, casein, lactose,
mineral salts (sodium, calcium, potassium, and phosphorous) and
vitamins. It is poor in iron.

Suckling generates nerve impulse

Stimulates posterior Nerve impulse


pituitary to release is sent to
oxytocin hypothalamus

Pectoralis major muscles


Impulse
Contraction of inhibits prolactin inhibiting
myoepithelial cells of Fatty tissue hormone and stimulates
mammary glands prolactin releasing
hormone
Lobules

Rib Areola
Nipple
Milk forced Prolactin secretion
Milk duct
increases from
into ducts anterior pituitary
Skin

Chest Blood vessel


wall

Suckling by Circulates to alveoli


the baby and promotes
Fig. 3.2.3 Mammary glands lactation

07
Biology

3.3 GAMETOGENESIS
‹ Gametogenesis is the process of production of gametes from the male and female primary
sex organs (gonads). It consists of three phases: Multiplication phase, growth phase, and
maturation phase.

Spermatogenesis Oogenesis

Process of formation of sperms Process of formation of ovum

2 stages: Foetal life- Formation


2 stages: Formation of of primary oocyte and adult/
spermatids and spermiogenesis reproductive life- Formation of
ovum

‹ The primary male sex organ is the testis and the female sex organ are the ovaries.
‹ The male gamete is the sperm and the female gamete is the ovum.

3.3.1 SPERMATOGENESIS

Type A
spermatogonia
Mitosis
Type B
Type A spermatogonium
spermatogonium

Primary
spermatocytes

Secondary
spermatocytes
Meiosis II

Spermatids
(2 stages of
differentiation)

Spermiogenesis

Spermatozoa
Lumen

Fig. 3.3.1 Representation showing spermatogenesis in the seminiferous tubules

08
Human Reproduction

At puberty in males, Sperms are finally


spermatogonia (germ cells) released in the lumen
cells present in seminiferous 1 7 of seminiferous tubules
tubules of the testis produce by the process of
sperms. spermiation.(NEET 2018)

Spermatogonium is a diploid Then sperm heads get


2 6 embedded in the sertoli cells.
cell with 46 chromosomes.
(NEET 2015)
Spermatids undergo process
Some spermatogonium develop as 3 5 of transformational change
primary spermatocytes (diploid), (spermiogenesis) to produce
and undergo meiosis I to form spermatozoa (sperms).
secondary spermatocytes (haploid/
23 chromosome) .(AIPMT 2008) 4
Secondary spermatocyte
undergoes meiosis II to form 4
haploid cells called spermatids

Flowchart showing the process of spermatogenesis

‹ The whole process of spermatogenesis is under regulation by the hypothalamic hormone,


gonadotropin releasing hormone (GnRH), which stimulates the production of the follicle
stimulating hormone (FSH) and luteinising hormone (LH).

Stimulates
LH stimulates FSH acts on
GnRH level anterior pituitary
Leydig cells Androgens initiate Sertoli cells to
increases at to release
to release spermatogenesis support the
puberty gonadotropins-
androgens spermiogenesis
LH and FSH

Flowchart showing hormonal control of spermatogenesis

‹ The male gamete, sperm, is divided into a head, neck, mid piece, and tail. The complete
sperm is covered by the plasma membrane. It is 0.06 mm long.

Tail Midpiece Head

Neck Nucleus
Axial filament Mitochondrion

Proximal centriole Acrosome

Fig. 3.3.2 Structure of a sperm

09
Biology

• Is oval in shape

• Has an elongated

haploid nucleus
(Anterior or first part)

(consists of
chromosomes) • Has axial filaments

Middle piece


• Has an acrosome • Helps in the move-


(cap-like structure); it • Has mitochondria in ment of sperm in
Head


is the anterior portion large numbers the fluid medium

Tail
of the head filled with • Provide energy for • The tail has a last



lytic enzymes that sperm motility part as an end piece
help in fertilisation which has only

(AIPMT 2010) naked filament

• Acrosome is formed

by Golgi bodies
• Neck is present

behind the head

‹ The secretions of the accessory ducts are essential for the maturation and motility of sperms.
‹ Semen consists of seminal plasma and sperms.
‹ A healthy male ejaculates around 200-300 million sperms during coitus.

Beyond NCERT
Mitochondrial formation in the sperms of some insects is called
Nebenkern sheath.

3.3.2 OOGENESIS
Primary Primary Developing
Blood Ovum
vessels Follicle Follicles Secondary Ovum
Primordial
Follicle

Graafian
Follicle

Corpus
Albicans

Ruptured
Follicle
Corpus Luteum

Germinal Epithelium
Developing Tunica Albuginea Liberated Ovum
Corpus Luteum
Fig. 3.3.3 Cross section representation of ovary

10
Human Reproduction

Corona radiata
Zona pellucida

Nucleolus
Nucleus

Cytoplasm
Secondary oocyte completes
Plasma meiosis-II in the fallopian tube
membrane only if fertilisation occurs.


(NEET 2020, AIPMT 2015)


Fig. 3.3.5 Structure of an ovum

‹ Ovum is a spherical structure with a diameter of 0.2 mm. It has three membranes- outer
corona radiata formed of follicle cells, the middle is the zona pellucida, and the innermost
layer is the plasma membrane. (NEET 2020)

Oogenesis Follicle development

Oogonium Primordial
Mitosis follicle

Primary oocyte
arrested in Primordial
prophase I follicle
Before birth
Childhood - ovary inactive
From puberty to menopause
Primary
follicle
Primary oocyte
Growing
Meiosis I follicle

Mature
Secondary oocyte follicle
First polar
body (dies)

Secondary oocyte, arrested


Ovulation
in metaphase II, Ovulated
Meiosis II
(completed only
if fertilized)

Zygote Corpus
luteum
Second polar
body (dies)
Fig. 3.3.4 Representation showing oogenesis in the ovary

11
Biology

Oogenesis starts at the female fetal stage; millions of oogonia (germ cells/
gamete mother cells) form in each ovary. It is a pre-birth process during the fetal
development.

Oogonia (46 chromosomes) start dividing in the fetus; enter into prophase-I of
meiosis, and stops division there. These cells are called the primary oocytes
(diploid).

Primary oocyte then gets enveloped by granulosa cells and is called the primary
follicle.

Out of a million primary follicles, only 60,000-80,000 follicles survive in each ovary
till puberty.

At this stage, the primary follicles develop more granulosa layers and new theca,
and then are called secondary follicles. Later it transforms into the tertiary follicle
(having fluid-filled cavity antrum).

Tertiary follicle has primary oocyte that grows in size and completes meiosis I,to
produce a large secondary oocyte (23 chromosomes/ haploid) and a first tiny polar
body. The secondary oocyte enters into meiosis II and gets arrested at metaphase-II.

At this stage, the tertiary follicle changes into a mature follicle or a Graafian
follicle. (NEET 2013)

Secondary oocyte (at metaphase-II) inside the Graafian follicle develops new layer
around it called the zona pellucida.

Graafian follicle then ruptures to release the secondary oocyte (ovum) from the
ovary (this process is called ovulation).

Secondary oocyte completes meiosis II during fertilisation to release ovum


(haploid) and second polar body; polar bodies degenerate.

1. During spermatogenesis, one spermatogonium produces four




spermatozoa (sperms), while one spermatid develops into one NCERT


sperm. One primary spermatocyte produces 4 sperms and one Highlights

secondary spermatocyte produces 2 sperms.


2. During oogenesis, one oogonium produces only one ovum.

12
Human Reproduction
Phases of gametogenesis

Fig. 3.3.6 Diagram showing comparative analysis of spermatogenesis and oogenesis

TRAIN YOUR BRAIN


Question : How many spermatozoa will be produced from 50
primary spermatocytes, and how many ova will be produced
from 50 primary oocytes?
Solution : One primary spermatocyte produces 4 sperms by
the process of spermatogenesis, while one primary oocyte
produces only one ovum by the process of oogenesis.
Therefore, 50 primary spermatocytes will produce 200 sperms (50 × 4), and 50 primary
oocytes will produce 50 ova.

13
Biology

3.4 MENSTRUAL CYCLE


‹ It is the cycle of events in human females and other primates (monkeys, apes) that bring out
changes in the ovary and uterus of the female. It is controlled by the endocrine hormones.
‹ The start of the menstrual cycle at puberty (14-15 years) is termed as menarche and the
end of the cycle at the age of 45-50 years is called menopause.

It is a repetitive cycle; one


Cycle starts from the
cycle is completed in 28-29
menstruation
days

One cycle has 4 phases-


the Menstrual phase
It is an indicator of healthy
Ovulation occurs in the (bleeding phase), follicular
reproductive phase of the
middle of each cycle phase (proliferative phase),
female
ovulatory phase, and luteal
phase (secretory phase)

Flowchart showing characteristics of the menstrual cycle

‹ Period from puberty to menopause is called reproductive period of the female life.

Phase Days Events Hormonal changes


Menstrual 1st - 5th Endometrium breaks; bleeding Progesterone decrease;
phase (menstruation) begins; ovum estrogen decreases
remains unfertilised (NEET 2013)
(NEET 2018)
Follicular 6th - 13th Endometrium rebuilds FSH and estrogens secretion
phase begins and starts increasing
Ovulatory 14th day (mid Ovulation occurs FSH and LH at peak;
phase of the cycle) estrogen is also at high level
(NEET 2020)
Luteal phase 15th - 28th Endometrium thickens; uterine Progesterone secreted by the
or secretory (AIPMT 2012) glands become secretory corpus luteum (NEET 2014)
phase

Table 3.4.1 Summary of the four phases of the menstrual cycle

14
Human Reproduction

Phases

Gonadotropic
Hormone Levels

Ovarian
cycle

Ovarian
Hormone
Levels

Uterine
cycle

Phases of the
Uterine Cycle

Days 1 7 14 21 28
Fig. 3.4.1 Diagram showing events of the menstrual cycle

Menstrual phase
• Endometrium lining of the uterus breaks down and menses occurs.


• Decrease in the level of LH (luteinising hormone) leading to a




degeneration of the corpus luteum.


• Decrease in the level of progesterone.


• GnRH levels are controlled by estrogen and progesterone. (NEET 2016)




Follicular phase
• Level of gonadotropins (FSH and LH) increase stimulating the


development of primary follicle to the Graafian follicle. (NEET 2016)




• Estrogen is released by the growing follicles.




• Endometrium starts re-building.




15
Biology

Ovulatory phase

• Levels of FSH and LH are at their peak



• LH surge (NEET 2015)


• Rupture of the Graafian follicle and release of the ovum (ovulation)

Luteal phase or Secretory phase
• Ruptured Graafian follicle develops as the corpus luteum

• Secretion of progesterone by the corpus luteum

• In absence of fertilisation, corpus luteum degenerates and becomes

corpus albicans. Progesterone level decreases; disintegration of the
endometrium marking the beginning of a new menstruation cycle

‹ In the presence of fertilisation, the corpus luteum does not degenerate and level of
progesterone remains same.The continuous release of progesterone is essential for the
maintenance of the endometrium and implantation of the fertilised ovum.
‹ During pregnancy, there is no menstruation.

Beyond NCERT
Relaxin and inhibin are two hormones that are also released by
the corpus luteum. Relaxin inhibits the contraction of the uterine
muscles and helps in the later stages of pregnancy. Inhibin inhibits
the secretion of FSH and LH.  (NEET 2016)
Corpus luteum is also called the yellow body as it is yellow in
colour due to the pigment lutein.

3.5 FERTILISation AND IMPLANTATION


‹ Fertilisation is the fusion of the haploid male gamete (sperm) with the haploid female gamete (ovum).
Sperm nucleus Zona pellucida

Cytoplasm of ovum

Follicle
cell

Acrosome

First polar body


Egg nucleus

Fig. 3.5.1 Diagram showing fertilisation of egg

16
Human Reproduction

NCERT
The discharge of semen into the female’s vagina is called Highlights
insemination.

Sperm reaches to the fallopian tube by swimming through the uterus

Fertilisation occurs, if the ovum is also released at the same time

Fertilisation occurs in the ampullary region of the fallopian tube (NEET 2016)

Sperm makes contact with the zona pellucida of the ovum

Only one sperm enters the ovum, as acrosome induces changes in the membrane
of the ovum to block entry of more sperms

Lytic enzymes of the acrosome helps in the entry of sperms into the ovum

Entry of sperm marks the completion of meiosis II in the secondary oocyte to


release haploid ovum (ootid) and the second polar body (NEET 2019)

Nucleus of ootid and sperm fuse to form a diploid zygote

Flowchart showing events of fertilisation

Beyond NCERT
1. The activation of sperm occurring in the female genital


tract by the secretions of the female genital tract is called


capacitation.  (NEET 2017)
2. Sperm has several lytic enzymes in the acrosome called as


sperm lysins. These are:


(a) Corona penetrating enzyme: Dissolves corona radiata.


(b) Zona lysine or acrosin: Digests zona pellucida.




(c) Hyaluronidase: Acts on follicle cells.




3. At the point of entry of sperm into the egg, a projection forms, which is called


fertilisation cone or cone of the reception.

17
Biology

Sex determination of the embryo


‹ It gets decided at the fertilisation stage.
‹ Human female has the XX sex chromosome, and the human male has the XY sex
chromosome.
‹ So, all haploid ovum has X chromosomes, and sperms have either X or Y chromosomes.
Therefore, the gametic fusion will produce either XX zygote or XY zygote, depending on the
condition, whether the X chromosome of sperm fuses with the X chromosome of ovum or
the Y chromosome of sperm fuses with X chromosome of ovum.
‹ XX zygote will develop into the female and the XY zygote will develop into a male. Hence,
the presence of the Y-chromosome of sperm will determine the sex of the baby.
‹ Zygote is a diploid cell with 46 chromosomes. It is a single cell that divides further to form
an embryo. Fertilisation is followed by a series of rapid mitotic divisions of the zygote called
cleavage.

After fertilisation, the cleavage starts in the zygote as it moves through the isthmus
of the fallopian tube (oviduct) towards the uterus for implantation

Cleavage occurs in the zygote to form the 2, 4, 8, 16 celled stages. These daughter
cells are called blastomeres

The 8 to 16 blastomeres stage is called the morula. It is a solid mass of cells that
looks like a mulberry fruit

The morula transforms into a blastocyst (blastula) which is a 64 celled stage

The blastocyst develops a cavity called blastocoel in it, and its cells get arranged
into the outer layer of cells called the trophoblast and inner cell mass

Embryo at this stage gets implanted (attached) to the endometrium of the uterus
through the trophoblast and uterine cells cover it

The inner cell mass starts developing as an embryo

The implantation of the embryo in the uterine wall leads to pregnancy

Flowchart showing stages of growth of the embryo

18
Human Reproduction

Morula
4 cell Stage 8 cell Stage (72Hours)

2 Cell Stage
Early Blastocyst
(48Hours)

Blastocyst
Zygote (4 Days)

Implanted
Blastocyst

Fertilization

Ovum

Fig. 3.5.2 Diagram showing transport of ovum, stages of growth of embryo in the fallopian tube

3.6 PREGNANCY AND EMBRYONIC DEVELOPMENT


‹ The further development of the embryo and progression of pregnancy is marked by the
formation of placenta.
‹ Placenta is the structural and functional unit that establishes a common connection between
the foetus and the mother. It is formed by the interdigitation of the chorionic villi and uterine
tissues.
‹ Chorionic villi are the finger-like projections of the trophoblast that are grown into the tissues
of the uterus wall.
‹ Placenta is connected to the embryo through the umbilical cord that helps in the transport of
nutrients and other substances to and from the embryo.

19
Biology

Fig. 3.6.1 Diagram representing human foetus in the ovary

Beyond NCERT
1. Amnion is the extra-embryonic or the fetal membrane that is lined


with an ectoderm and covered with mesoderm. (NEET 2018) 

2 Ectopic pregnancy is the implantation of the embryo at the site




other than the uterus.  (NEET 2015)

Act as a medium of transport between


mother and foetus
Functions of the

Supplies oxygen and nutrients to the foetus


placenta

Takes carbon dioxide and waste from the foetus to


remove them

Acts as a barrier between the mother and


the foetus

Secretes hormones like estrogens, progesterone, hCG (human


chorionic gonadotropin) and hPL (human placental lactogen)

Flowchart showing functions of the placenta

‹ During pregnancy, the female body has an increased secretion of estrogens, progestogen,
cortisol, prolactin, and thyroxine which is essential for the growth of the embryo.

20
Human Reproduction

Heart is formed after one month of fertilisation

Limbs and digits are formed by the end of the second month

The major organs like heart, liver, kidneys, and genital organs gest well developed
by the end of the first trimester (first 3 months or 12 weeks)

Foetus develops first movement and hair on the head during the 5th month

Body gets covered by fine hair by the end of second trimester (24 weeks). The
eyelids separate, and eye lashes also appear by this time

The foetus is ready for delivery by the end of 9 months

Flowchart showing features of embryonic development by the months of pregnancy

During development, the inner cell mass gets differentiated into three germ
layers: Outer layer is the ectoderm; middle layer is the mesoderm; inner layer NCERT
is the endoderm. This germ layered structure is called the gastrula the process Highlights

of development of these layers is called gastrulation. These germ layers have


the capacity to develop all the tissues and organs of the body (stem cells).

3.7 PARTURITION AND LACTATION


‹ Gestation period is the average duration of pregnancy (period from conception to birth). It
is approximately 9 months in humans.
‹ Parturition is the process of delivery of the foetus at the end of pregnancy. It is also called
childbirth.

Parturition Lactation
• Process controlled by neuroendocrine • The first milk produced by


mechanism the mother after the birth is
• Foetal ejection reflex- Signals induced by the called colostrum, which is

placenta and the foetus to initiate uterine the yellowish milk and full of
contractions for child birth (AIPMT 2012) nutrients and antibodies, which

• The reflex stimulates the release of oxytocin develops resistance in the new-

from the pituitary of the mother (NEET 2015) born baby
• Oxytocin increases uterine contractions to • Lactation is the process of


many fold and continue its secretion till the production of milk by the
childbirth through the birth canal mammary glands towards the
• Umbilical cord and the placenta are removed end of pregnancy and after

after the birth childbirth.

21
LEVEL - II
Human Reproduction
Introduction
Humans are sexually reproducing and viviparous organisms. Their reproductive events include: -
• Formation of gametes (gametogenesis), i.e., sperms in males and ovum in females.
• Transfer of sperms into the female genital tract (insemination) and fusion of male and female
gametes (fertilisation) leading to formation of zygote.
• This is a followed by formation and development of blastocyst and its attachment to the uterine
wall (implantation), embryonic development (gestation) and delivery of the baby (parturition).

These reproductive events occur after puberty. There are remarkable differences between the
reproductive events in the male and in the female, for example, sperm formation continues even in old
men, but formation of ovum ceases in women around the age of fifty years.

Primary sex organ:


Essential organs which form the gametes and also secretes sex hormones. In males, the gamete
forming organ are the testes. In females, the corresponding organ are ovaries. The male gamete is the
spermatozoan. The female gamete is the ovum.

Secondary sex organ:


• These form the passage for the gametes to help the union of male & female gametes.
• In male this includes epididymis, vas deferens, seminal vesicles, prostate, bulbourethral glands &
penis
• While in female - Fallopian tube, uterus & vagina
• Breast is an accessory sex organ.

Development of sex organ:


• During intra uterine life (IUL) testis & ovary develop from mesoderm. They develop in abdominal
cavity in IUL, at the time of birth, testes descend down into scrotal sac but ovaries remain in
abdominal cavity.

Male Reproductive System

• The male reproductive system is located in the pelvic region.


• It includes pair of testes along with accessory ducts, glands and the external genitalia.
• Both testes are situated out side the abdominal cavity with in a pouch called as scrotum or scrotal
sac so the testes of human males are extra-abdominal.
• The temperature of scrotum is 2–2.5°C lesser than body temperature. It is necessary for
spermatogenesis.
• Internally scrotum is lined by dartos muscle & spermatic fascia. Dartos muscle helps in regulation
of the temperature with in the scrotum during cold season, It becomes contracted in cold & during
warm season, it becomes relaxed.
• Cremaster muscles line inside the wall of scrotal & inguinal canal region and help in elevation of
testes.

22
• Each testis is attached to the walls of the scrotal-sac through flexible, elastic fibres. This group of
fibres is called Gubernaculum.
• Each testis is attached to the dorsal body wall of the abdominal-cavity through a cord termed as
the Spermatic-cord. This cord is made up of elastin fibres & spermatic fascia. The contents of cord
are vas deferens, gonadal veins, gonadal arteries, nerves and lymphatics.
• During embryonic stage, testes develop in abdominal cavity & they descend to reach the scrotum.
When the testes does not descend to reach the scrotum but remain in abdominal cavity at the
time of birth this condition is called undescended testes or cryptorchidism. Such testis can not
develop and function properly and may develop malignancy.
• Each scrotum is connected to the abdominal cavity through a passage termed as inguinal-canal.
Through this canal the testis descend down into the scrotal-sacs at the time of birth.
• Spermatic cord in males passes through the inguinal canal.
• Orchiopexy: When the undescended testes are brought into scrotal sac by surgical process during
childhood this process called as orchiopexy.

Fig. Diagrammatic view of male reproductive system


(part of testis is open to show inner details)
Internal Structure of Testis:
• Each testis is oval in shape with a length about 4 to 5 cm and a width of about 2 to 3 cm.
• Testis is covered by three coats. Outer most is tunica vaginalis. Middle coat is tunica albuginea &
inner most is tunica vasculosa.
• Tunica vaginalis has a parietal & visceral layer. It covers the whole testis except it's posterior border
from where the testicular vessels & nerves enter the testis.
• The Tunica albuginea is a dense, white fibrous coat covering the testis all around. The posterior
border tunica albuginea is thickened to form vertical septum called the Mediastinum testis.
• Tunica vasculosa is the inner most vascular coat of the testis lining testicular lobules.
• Each testis has about 250 compartments called testicular lobules.
• Each lobule has 1 to 3 highly coiled seminiferous tubules, in which sperms are produced.
• Total number of seminiferous tubules in each testis is about 750 to 1000.

23
Internal structure of Seminiferous Tubule:

Fig. Sectional view of seminiferous tubule


• Outer surface of seminiferous tubules is composed of white fibrous connective tissues called as
tunica propria. While inner surface is of cuboidal germinal epithelium.
• In germinal epithelium two types of cells present (i) male germ cells (spermatogonia) (ii) Sertoil
cells.
• Spermatogonia give rise to spermatozoa which are released into the lumen of the tubule. In
between spermatogenic cells, Sertoil or sustentacular or nurse cells are present which provide
nourishment to developing spermatozoa.

Functions of Sertoli cells:


• To release Androgen Binding Protein (ABP)
• To form blood-testis barrier.
• Regulate spermatogenesis activity by releasing inhibin to check FSH over activity.
• The regions outside the semniferous tubules called interstitial spaces, contain small blood vessels
and groups of cells called Interstitial cells or Leydig cells, are located in the connective tissue
around the seminiferous tubules. They constitute the endocrine tissue of the testis. Leydig cells
synthesis and secrete testicular hormones called Androgens into the blood. Other immonologically
competent cells are also present.

24
• Tubuli recti, rete testis and Vasa effferentia from the intra testicular genital duct system.

• These vasa efferentia come out from upper dorsal surface of testis & open into epididymis. This
epididymis is responsible for functional maturation of sperm. It is located along the posterier
surface of each testis.
Vas efference and epididymis has Stereociliated inner lining.

Epididymis has 3 parts:


(i) Upper, highly coiled part - Caput epididymis
(ii) Middle part - Corpus epididymis
(iii) Basal, least coiled part - Cauda epididymis
• Cauda-epididymis, enters inside the abdominal-cavity from the scrotal-sac in the form of Vas
deferens or ductus deferens. Terminal dilated part of vas deferens are called ampulla.

• Vas deferens runs upward & enter into abdominal cavity. Both vas deferens coil around the
ureter of their respective sides and then dilate to form ampulla. Ampulla of each side receives
the duct of seminal vesicle of that side and forms ejaculatory duct and opens into prostatic
urethra.

• The male accessory ducts include rete testis, vasa efferentia, Epididymis and vas deferens.

Urethra:

• The urethra originates from the urinary bladder and extends through the penis to its external
opening called urethral meatus.
• Male urethra provides a common pathway for the flow of urine and semen. It is much longer in
male than in female, measuring about 20 cm.
Male urethra is differentiate into 3 parts:-
(i) Prostatic urethra is surrounded by prostate gland.
(ii) Membranous urethra is the second part which is situated behind the lower part of pubic
symphysis and is smallest.
(iii) Penile urethra is situated in the penis and is the longest part.
The urethra receives the ducts of the prostate and Cowper’s glands, passes through the penis
and opens to the outside.

Path of Sperm through the Male Body:

25
Fig. Diagrammatic sectional view of male pelvis showing reproductive system

Penis:
• The Penis is the male external genitalia or copulatory organ of man.
• Terminal part of penis is bulging, it is called as Glans penis. This glans penis is covered by a movable
skin called as prepuce or foreskin.
• It is made up of special tissue that helps in erection of the penis to facilitate insemination.
• Penis is composed of three longitudinal cylindrical cords of erectile tissue. These cords are the
right & left corpora cavernosa & a median corpus spongiosum. The two corpora cavernosa do not
reach the end of the penis. Each of them terminates under cover of the glans penis. The corpus
spongiosum continues further, its terminal part is expanded to form a conical enlargement called
the glans penis. Urethra runs through this cord. External opening of penis is called penile/urethral
meatus.
• Erection of Penis: Erection of penis is purely vascular phenomenon and is controlled by A.N.S.
It occurs due to increase of blood supply, due to dilation of penile arteries causing enlargement
and hardening of penis. During this time the muscles of Penis are relaxed.

Accessory Reproductive Glands:

The substances secreted by the accessary reproductive glands help in reproduction, these are –
1. Seminal vesicle: These are paired, tubular, coiled glands situated behind the bladder. Internally,
it is lined by glandular epithelium which secretes seminal fluid, which is lubricating, transparent
& jelly like substance, which makes 60–70% part of semen. It is slightly alkaline (pH 7.3).
Fructose is found in seminal fluid, it act as fuel to sperm. Fibrinogen, and several proteins are
also present in semen.

2. Prostate gland: This gland is unpaired and located below the urinary bladder. It secretes
prostatic fluid which is milky, thick, sticky or jelly like. It makes about 30% part of semen and
helps in sperm activation.
In the secretion of prostate-gland Calcium and clotting enzyme and profibrinolysin are present.
The secretion of the prostate gland combines with the secretion of seminal vesicle and so the
semen gets coagulated. In the coagulated semen, the mobility of sperms is reduced and so their
energy is conserved. After some time due to fibrinolysins, semen again liquefies and in this
semen now the sperms can move.

3. Cowper's glands: It is a pair of glands found on lateral side of urethra. It is also called as
bulbourethral gland. It is situated in membranous part of urethra & opens into penile urethra.
It secretes transparent, slimy, jelly like fluid. It is slightly alkaline (pH is 7.2). This destroys the

26
acidity of the urethra and cleans it for the movement of sperms. The secretions of bulbourethral
glands also helps in the lubrication of the penis.

Semen – Semen = Sperm + Accessory reproductive gland fluid (Seminal Plasma)


Volume = 3 to 4 ml.
pH = 7.3 - 7.5 (Slightly alkaline)
Oligospermia < 20 million/ml.
Azospermia – either absence or near absence of sperms.
Male human ejaculates about 200 to 300 million sperms during a coitus of which, for
normal fertility, at least 60 percent sperms must have normal shape and size and for at
least 40 per cent of them must show vigorous motility.
• Secretions of these glands constitute the seminal plasma which is rich in fructose, calcium
and certain enzymes.

Concept Builder

1. The male sex accessory ducts do not include –


(1) Rete testis (2) Vasa efferentia (3) Epididymis (4) Ureter

2. Which of the following structure is paired –


(1) Ejaculatory duct (2) Prostate (3) Urethral meatus (4) Urethra

3. Epididymis is located in ____ of testis.


(1) Anterior surface (2) Posterior surface
(3) Lateral surface (4) Anterolateral surface

4. Function of bulbourethral gland secretion is –


(1) Nourishes sperms (2) Form major part of semen
(3) lubrication of Penis (4) Enhancing the motility of sperms

5. Tubuli recti of semniferous tubules open into –


(1) Epididymis (2) Vasa efferentia
(3) Vasa deferentia (4) Rete testis

6. Common duct formed by the union of vas deferens & duct of seminal vesicle is –
(1) Urethra (2) Ureter
(3) Ejaculatory duct (4) Spermatic duct

7. Sperms are stored & nourished inside –


(1) Cowper's gland (2) Epididymis
(3) Semniferous tubules (4) Vasa efferentia

8. Role of leydig cells of tesis is –


(1) Provide nourishment to sperms (2) Provide motility to sperms
(3) Bring about maturation of sperms (4) Synthesis of testosterone hormone

9. Vas deferens arises from –


(1) Cauda epididymis (2) Caput epididymis
(3) Corpus epididymis (4) Rete testis

Concept Builder (Answer-Key)


Que. 1 2 3 4 5 6 7 8 9
Ans. 4 1 2 3 4 3 2 4 1

27
Female Reproductive System
The female reproductive system consists of a pair of ovaries, a duct system consisting of a pair of
fallopian tubes (oviducts), a uterus, cervix and vagina. A pair of mammary glands are accessory genital
glands.

Fig. Sectional view of the female reproductive system


Ovaries:
• The ovary is the primary female sex organ. It produces ova and secretes the female sex hormones,
estrogens and progesterone which are responsible for the development of secondary female sex
characters and cause marked cyclic changes in the uterine endometrium. The human ovaries are
small about 2 to 4 cm in length and is connected to the pelvic wall and uterus by ligaments.
• Each ovary is covered by a thin epithelium which encloses the ovarian stroma. The stroma is divided
into two zones – a peripheral cortex and an inner medulla.

Fallopian Tubes (Oviducts):


• These are one pair of long (10 to 12 cm), ciliated muscular and tubular structures which extend
from the periphery of each ovary to the uterus.
• Each oviducts is differentiated into three parts:
(i) Infundibulum: The part of oviduct closer to the ovary is the funnel shaped infundibulum. The
edges of infundibulum possess finger-like projections called fimbriae. Fimbriae help in the
collection of the ovum after ovulation. Infundibulum opens into the abdominal cavity by an
apperture called ostium.
(ii) Ampulla: The infundibulum leads to a wider part of the oviduct called ampulla.
(iii) Isthmus: It is the last and narrow part having narrow lumen that links to the uterus.

• This tube is involved in conducting of ovum or zygote towards the uterus by peristalsis and ciliary
action. It is also the site of fertilization. (Fertilization occurs in ampulla).
Uterus (Hystera/Womb):

• It is a large hollow, muscular, highly vascular and inverted pear shaped structure present in the
pelvis between the urinary bladder and rectum.

28
• It has the following three parts: -
(i) Fundus: It is upper, dome-shaped part above the opening of fallopian tubes.
(ii) Corpus/Body: It is the middle and main part of uterus.
(iii) Cervix: It is lower, narrow part which opens in body of uterus by internal os and in vagina
through external os. The cavity of cervix is called cervical canal which along with vagina forms
the birth canal.

• Wall of uterus: The wall of uterus is formed of three layers: -


(i) Outer peritoneal layer, perimetrium.
(ii) Middle muscular myometrium of smooth muscle fibres.
(iii) Inner highly vascular and glandular endometrium.
• The endometrium undergoes cyclic changes during menstrual cycle while myometrium exhibits
strong contractions during delivery of the baby.
• Implantation of embryo occurs in uterine fundus.
• It is the site of foetal growth during pregnancy. It also takes part in placenta formation and
expulsion of the baby during parturition.

Fig. Diagrammatic sectional view of female pelvis showing reproductive system

Vagina:
• It is a long fibro-muscular tube. It extends backward in front of rectum and anal canal from cervix
to the vestibule. It is a highly vascular tube lined internally by mucus membrane which is raised
into transverse folds called vaginal rugae. It is lined with stratified squamous epithelium (Non
Keratinised).
• Vagina acts both as copulation canal (as it receives the sperms from penis during copulation) and
as birth canal along with cervix (during parturition).
External Genitalia/Vulva :
External genitalia of female include mons pubis, labia majora, labia minora, hymen and clitoris.

1. Mons pubis: It is a cushion of fatty tissue covered by skin and pubic hairs in adult female.

2. Labia Majora: Fleshy folds of tissue, which extend down from the mons pubis and surround the
vaginal opening. It is homologous to the scrotum.

3. Labia Minora: Paired folds of tissue under the labia majora.


4. Clitoris: A tiny finger-like structure which lies at the upper junction of the two labia minora above
the urethral opening. Clitoris is a homologous to the penis in the male.

29
5. Hymen: The opening of the vagina is often covered partially by a membrane called hymen.
The hymen is often ruptured during the first coitus (intercourrse). However, it can also be broken
by a sudden fall or jolt, insertion of a vaginal tampon, active participation in some sports like
horseback riding, cycling etc. In some women the hymen perists even after coitus, in fact, the
presence or absence of hymen is not reliable indicator of verginity or sexual experience.

Bartholin Glands:
• It is similar to Cowper’s gland of male.
• 1 pair Bartholin gland is found on lateral side of vagina and bartholin duct opens into vaginal
vestibule.

Mammary Glands

Fig. A diagrammatic sectional view of Mammary gland


• There are a pair of rounded prominences present over the pectoralis major muscles on the
front wall of the thorax. These remain in rudimentary form in male. In females, these remain
undeveloped till puberty. At puberty, these start developing under the influence of oestrogen
and progesterone hormones. In the external side, each breast has a projection, the ‘nipple’
surrounded by rounded hyperpigmented are called areola and appear deep pink or light brown.
On the surface of the areola, numerous sebaceous glands, called areolar glands are present.
Note:
• Each breast contain glandular tissue and variable amount of fat. The glandular tissue comprises
about 15-20 lobes in each breast. Each lobule is composed of grapelike clusters of milk secreting
cell termed alveoli. The cells of alveoli secretes milk which is stored in the cavity of alveoli.
When milk is produced it passes from the alveoli in the mammary tubules and then into the
mammary ducts. Near the nipple, mammary ducts expand to form mammary ampulla (=
lactiferous sinuses) where some milk may be stored before going to lactiferous ducts. Each
lactiferous duct typically carries milk from one of the lobes to exterior.
Mammary alveoli → Mammary Tubule → Mammary Duct → Mammary Ampulla →
Lactiferous Duct

30
Concept Builder

1. Bartholin's glands are situated –


(1) On medial side of vagina (2) On lateral side of vagina
(3) On lateral side of clitoris (4) On medial side of labia majora
2. The female external genitalia include –
(i) Ovary (ii) Mammary gland (iii) Mons pubis
(iv) Clitoris (v) Labia majora
(1) (i) & (ii) (2) (ii) & (iii) (3) (iii), (iv) & (v) (4) (ii), (iii) & (v)
3. Lower narrow end of uterus is called –
(1) Urethra (2) Cervix (3) Clitoria (4) Vulva
4. Ovaries connected to the pelvic wall & uterus by –
(1) Ligaments (2) Connective layer
(3) Epithelium layer (4) Muscular filament
5. The wall of the uterus is made of _____ layer –
(1) 3 (2) 2 (3) 1 (4) 4
6. It is a middle thick layer of smooth muscle, which exhibits strong contraction during delivery
of the baby –
(1) Endometrium (2) Myometrium (3) Epimetrium (4) Perimetrium
7. Which part of vulva is considered equivalent or homologous to the male penis –
(1) Clitoris (2) Hymen (3) Labia minora (4) Mons pubis

8. During puberty stage, which sex hormone stimulate the enlargement of breast –
(1) FSH & LH (2) Estrogen (3) Testosterone (4) Androgens

Concept Builder (Answer-Key)


Que. 1 2 3 4 5 6 7 8
Ans. 2 3 2 1 1 2 1 2

Gametogenesis
• Formation of gametes is called gametogenesis. The primary sex organs the testis in males and the
ovaries in the females produce gametes sperm and ovum respectively.
• GnRH, FSH, LH regulates gametogenesis.
• Besides this hormone vitamin E is also essential for gametogenesis. Deficiency of vitamin E leads
to sterility. Vitamin A is also required for the formation of healthy gametes.

• Gametogenesis is divided in three stages:


(i) Multiplication phase (ii) Growth phase (iii) Maturation phase.

• As there are two types of gametes, the spermatozoa and ova, gametogenesis can be studied under
two broad headings: spermatogenesis and oogenesis. Spermatogenesis is the formation of
spermatozoa, whereas oogenesis is the formation of ova. Both spermatozoa and ova originate from
primordial germ cells or PGCs, which are extra-gonadal in origin.
• Spermatogenesis occurs in the seminiferous tubules of the testes and oogenesis occurs in the
follicles of ovary. Formation of gametes starts at puberty.

31
(a) (b)
Fig. Schematic representation of (a) Spermatogenesis; (b) Oogenesis

Spermatogenesis
• In testis, the immature male germ cells (spermatogonia) produce sperms by spermatogenesis that
begins at puberty.
• Mammalian testes contain seminiferous tubules and wall of seminiferous tubule is composed of
germinal epithelium. It contains some special types of cells called primordial germ cells and
these cells start spermatogenesis. On the basis of origin, primordial germ cells are extra
embryonic mesodermal. Besides these cells, germinal epithelium contains some large sized cell
called sertoli cells. Occurrence of sertoli cells is the unique feature of mammalian testis. Sertoli
cells provide nutrition to developing sperm. Sperms are embedded in cytoplasm of sertoli cells
and absorb nutrition. After maturation sperms comes out from sertoli cells and librate in
seminiferous tubules.

• After spermatogenesis the sperms heads become embedded in the sertoli cells and are finally
released from the seminiferous tubules by the process called spermiation-
• Liberation of sperms from testes is called semination.
• Liberation of sperms from body of male is called ejaculation .
• Mammalian sperms are transferred to vagina of female by the process called insemination.

• Sertoli cells form 'blood testes barrier' and protect the sperm from immune system of the body
because antibody may attack on haploid cells and destroy them. (Sperms are haploid and other
cells of body are diploid).
• Spermatozoa are formed in the wall of the seminiferous tubules of the testes. The various cell-
stages in spermatogenesis are as follows ( the number of chromosomes at each stage is given in
brackets)

32
A. The spermatogonia (type A) or germ cells (44+X+Y) divide mitotically, to give rise to more
spermatogonia of type A (spermatogenic lineage) and also spermatogonia of type B. Each
spermatogonia is diploid and contain 46 chromosomes. Type A act as stem cells.
B. The spermatogonia (type B) (44 + X + Y) enlarge, to form primary spermatocytes.
C. The primary spermatocytes (44 + X + Y) now divide by meiosis into two secondary
spermatocytes. This is the first meiotic division: it reduces the number of chromosomes to half
(reduction division). Secondary spermatocytes are haploid cells which have 23 chromosomes.
D. Each secondary spermatocyte has 22 + X or 22 + Y chromosomes. It divides to form two
spermatids. This is the second meiotic division and this time there is no reduction in
chromosome number
E. Transformation of spermatid into
sperm is termed spermiogenesis. A
spermatid is non-motile and heavy. It
has organelles like mitochondria, Golgi
bodies, centrioles, nucleus etc. During
spermiogenesis, the weight of gamete
is reduced along with the
development of locomotory structure.
Nucleus becomes compact forming
the major part of head of spermatozoa.
Golgi complex of spermatid gives rise
to acrosome. The two centrioles of the
spermatids become arranged one
after the other behind the nucleus.
Mitochondria from different parts of
spermatid get arranged in the middle
piece around axial filament. Much of
the cytoplasm of a spermatid is lost.
After spermiogenesis the spem heads
become embedded in the Sertoli cells,
and are finally released from the
seminiferous tubules by the process
celled spermiation.

Male Reproductive Hormones

FSH = Binds with FSH receptors attached to sertoli cells. FSH causes to grow and secrete various
spermatogenic substances and androgen binding proteins (ABP).

ABP: Concentrates the testosterone inside seminiferous tubules.(Androgens)

LH / ICSH: It stimulates the Leydig cells to secrete testosterone. Inhibin: It is secreted by Sertoli cells
in response to excess spermatogenesis. The inhibin gives a negative feedback to the hypothalamus and
anterior pituitary, this results in suppression of synthesis and release of FSH (so spermatogenesis
decreases).

33
Testosterone: Secreted by Leydig cells. It is essential for
(i) Stimulate spermatogenesis
(ii) Development of secondary sexual characters
(iii) ABP secretion
(iv) It also gives –ve feedback to hypothalamus and anterior pituitary in its excess concentration to
suppress GnRH, FSH & LH release.
(v) It is secreted in foetal stage in as low as 30 ng/ml plasma concentration to cause descent of testis
in last trimester of intrauterine life.

Structure of Sperm
• Mature sperm cell consists of a head, a neck, a middle piece and a tail.
• A plasma membrane envelops the whole body of sperm. The sperm head contains a very little
cytoplasm, an elongated haploid nucleus, the anterior portion of which is covered by a cap-like
structure, acrosome. Nuclear part of head of spermatozoa consist of chromatin (mostly DNA) that
is extremely condensed. The acrosome is filled with enzymes that help in fertilisation of ovum.
These enzymes called sperm lysins that dissolve the membranes enveloping the ovum and help
the sperm to enter the ovum. Acrosome is derived from golgi apparatus. Its membrane extends
down the outer surface of nucleus.
• The short neck, contains two distinct granules- the proximal and distal centrioles. The proximal
centriole plays a crucial role during the first cleavage of the fertilized ovum. The distal centtriole
gives rise to the axial filament of the long tail of the sperm.

34
Fig. Structure of a sperm
• The middle piece possess spiral sheath of numerous mitochondria (25 to 30 arranged spirally)
called Nebenkern sheath, which produce energy for the movement of tail that facilitates sperm
motility essential for fertilisation, that is why is called as the power house of the sperm.

• At the point where the middle piece joins the tail, this axial filament passes through a ring-like
structure called the annulus or ring centriole.

• The tail is made up of a central axial filament surrounded by a small amount of cytoplasm and cell
membrane as external sheath. Tail is the longest part of sperm. Sperm move by the help of tail.

Oogenesis
• The process of formation of a mature female gamete is called oogenesis. It is initiated during the
embryonic development stage when a couple of million gamete mother cell (oogonia) are formed
with in each foetal ovary, no more oogonia are formed after birth. Scattered ovarian follicles are
embedded in the stroma of cortex.

35
• Oogenesis process can be divided into three stages:
(A) Multiplication phase (B) Growth phase (C) Maturation phase
(A) Multiplication phase: In this stage primordial germ cells or ovum mother cells repeatedly divide
by mitosis to form large number of diploid oogonia.
This process completes in embryo stage of female in most higher animals.

(B) Growth phase: Like spermatogenesis, in this process oogonia grow in size and form primary
oocytes. The growth phase is the longest phase in oogenesis in oviparous animals. During growth
phase size of egg increases many times. In this phase yolk is formed.

(C) Maturation phase: It is the longest phase in human. By the time the foetus is 25 weeks old, all
the oogonia that she will ever produce, are already formed by mitosis. Hundreds of these diploid
cells develop into primary oocytes, begin the first steps of the first meiotic division, proceed
up to diplotene and then stop any further development. The oocytes grows much larger and
completes the meiosis I, forming a large secondary oocyte and a small polar body that receives
very little amount of cytoplasm but one full set of chromosomes.

36
• In humans (and most vertebrates), the first polar body does not undergo meiosis II, whereas the
secondary oocyte proceeds as far as the metaphase stage of meiosis II. However, it then stops
advancing any further, it awaits the arrival of the spermatozoa for completion of second meiotic
division. Entry of the sperm restarts the cell cycle breaking down MPF (M-phase promoting factor)
and turning on the APC (Anaphase promoting complex). Completion of meiosis II converts the
secondary oocyte into a fertilised egg or zygote (and also a second polar body)

• Ova are derived from oogonia present in the cortex of ovary.

• Some important differences between oogenesis and spermatogenesis are

(i) Whereas one primary spermatocyte gives rise to four spermatozoa, one primary oocyte forms
only one ovum.

(ii) When the primary spermatocyte divides, its cytoplasm is equally distributed between the two
secondary spermatocytes formed. However, when the primary occyte divides, almost all its
cytoplasm goes to the daughter cell which forms the secondary oocyte. The other daughter
cell (first polar body), receives half the chromosomes of the primary oocyte, but almost no
cytoplasm.

The first polar body is, therefore, formed merely to get rid of unwanted chromosomes.

Structure of Ovary

• Outer most layer of ovary is called germinal epithelium while the inner layer called Tunica albuginea
is made up of White fibrous connective tissue.

• The inner part of ovary is called as stroma. it is differentiated into 2 parts, outer peripheral part is

cortex & inner part is called medulla. Stroma consists of follicular cells, connective tissues, blood
vessels & lymphatics.

• Numerous oogonia are found in cortical region in intrauterine life. In early stage of intra uterine
life, they proliferate by mitosis, after which meiosis-I starts in them and proceeds upto prophase-
I stage & halts there itself up to puberty (when the ovulation starts). Now the halted meiosis-I
process restart at puberty causing primary oocyte to convert into secondary oocyte just before

ovulation. With this the Ist meiotic division completes and first polar body is formed. In secondary
oocyte immediately begins the second meiotic division but this division stops again at metaphase
stage. It proceeds further only when a sperm penetrates the oocyte.

37
Fig. Sectional view of ovary
Formation of ovarian or Graafian follicle:
Ova develop from oogonia present in the cortex of the ovary. The oogonia are surrounded by other cells
that form a stroma for them. These stromal cells form the ovarian or Graafian follicle that surrounds
the ovum and protects it.
The stages in formation of Graafian follicle are as follows:

(1) Firstly some cells of the stroma become flattened and surround a primary oocyte (which develops
from oogonia). These flattened cells ultimately form the ovarian follicle and are therefore called
follicular cells.
(2) The flattened follicular cells now become
columnar. Follicles upto this stage of Oocyte
development are called primordial follicle.
(3) A membrane called the zona pellucida, now Nucleus
appears between the follicular cells and the
oocyte. Follicular cell
(4) The follicular cells proliferate now to form
Stroma
several layers of cells to form the membrana
granulosa. These cells are now called
granulosa cells.
(5) As the follicle expands, the stromal cells surrounding the membrana granulosa become condensed
to form a covering called the theca Interna. The cells of theca internal (Thecal cells) afterwards
secrete a hormone called oestrogen.Outside the theca interna some fibrous tissue become
condensed to form another covering called the theca Externa. This is called secondary follicle.
(6) A cavity appears within the membrana granulosa. It is called the antrum. With the appearance of
this cavity, the Tertiary follicle is formed.
• Presence of antrum (Follicular cavity) is character feature of Tertiary follicle.

38
(7) The cavity of the Tertiary follicle rapidly increases in size and gets filled with a fluid called liquor
folliculi. Due to increase in the size of the cavity the wall of the follicle (formed by granulosa cells)
becomes relatively thin. The oocyte now lies eccentrically in the follicle, surrounded by some
granulosa cells that are called as cumulus oophoricus. The cells that attached to the wall of the
follicle are called as discus proligerus or Germ hill. The ovarian follicle is now fully formed and is
now called the Graafian follicle.

• The granulosa cells lying in the


close vicinity of the ovum
(secondary oocyte) and zona
pellucida, become elongated to
form the corona radiata.
• After 13 days of menstrual cycle
(on 14th day when cycle is ideally
for 28 days) Graafian follicle is
ruptured & egg is released.
• After ovulation the ruptured
Graafian follicle is called corpus
luteum. Soon after ovulation, the
granulosa cells of Graafian
follicle proliferate & these cells
look yellow due to accumulation
of pigment called Lutein. These cells are called lutein cells.
• Before ovulation the follicle was avascular but soon after ovulation blood vessels grow & corpus
luteum becomes filled with blood. Central part filled with blood is called corpus haemorrhagicum.
Lutein cells synthesis the progesterone hormone.
• If fertilization occurs in fallopian tube, the corpus luteum then becomes stable for next nine
months. If fertilization does not occur then the corpus luteum starts degenerating after about 9
days of it's formation. The degeneration is completed by 14 days to form corpus albicans, which
gradually disappears.

39
• Progesterone hormone maintains pregnancy and repairs the wall of uterus to make its surface
adhesive to help in implantation.
• The total number of follicles in the two ovaries of a normal young adult woman is about four lakhs.
A large number of these follicles degenerate from birth to puberty. Degeneration of ovarian follicle
is called follicular atresia and their disposal is done by phagocytes. Therefore, at puberty only
60000 to 80000 primary follicles are left in each ovary. Generally, only one ovum is liberated in
each menstrual cycle, by alternate ovaries. Only about 450 ova are produced by a human female
over the entire span of her reproductive life which lasts till about 40-50 years of age.
• In human female ovulation occurs in presence of FSH & LH. Coitus is not necessary for inducing
ovulation. Such a female is called as spontaneous ovulator. (Induce/Relax ovulator)
• Sometimes, two or more follicles reach maturity in one month or cycle, so more than one oocyte
may be ovulated. This is the commonest cause of multiple births. In such cases the sibling are
fraternal, not identical.

Concept Builder

1. Release of sperm from testes is called:


(1) Spermiation (2) Semination (3) Insemination (4) Ejaculation
2. At the end of first meiotic division, male germ cell differentiates into:
(1) Secondary spermatocyte (2) Primary spermatocyte
(3) Spermatogonium (4) Spermatid
3. How many ova are produced by 10 primary oocytes.
(1) 5 (2) 10 (3) 20 (4) 40
4. During which stage of gametogenesis meiosis occurs:
(1) Growth phase (2) Multiplication phase (3) Maturation phase (4) None of the above
5. How many autosomes does a human primary spermatocyte have:-
(1) 34 (2) 44 (3) 54 (4) 33
6. A complicated process of growth & change converts the spermatid into a:-
(1) Spermatogonia (2) Primary spermatocyte
(3) Secondary spermatocyte (4) Functional sperm or spermatozoa
7. Which is unique in the process of oogenesis as compare to process of spermatogenesis –
(1) Polar body formation (2) Meiotic division
(3) Equal cytoplasmic division (4) Mitotic division
8. How many ova are formed in oogenesis -
(1) Only one (2) Two (3) Three (4) Four
9. Which of the following is diploid ?
(1) Secondary spermatocytes, oogonia
(2) Spermatozoa & ova
(3) Spermatogonia, Oogonia, Primary spermatocyte
(4) Secondary oocytes, Primary Spermatocyte

Concept Builder (Answer-Key)


Que. 1 2 3 4 5 6 7 8 9
Ans. 2 1 2 3 2 4 1 1 3

40
Menstrual Cycle

• This is exhibited by primate group of animals. In this cycle the female body prepares itself for a
possible pregnancy. If the pregnancy does not occur then the body aborts all preparation done and
restarts the preparation for pregnancy again in a monthly cyclic manner.
• First menstruation begins at puberty and is called menarche.
• Menstrual cycle has three main phases:
(i) Bleeding phase or menstruation phase.
(ii) Proliferative/preovulatory/follicular phase or oestrogenic phase.
(ii) Secretory/post ovulatory/luteal phase or progesteronic phase.
(i) Bleeding Phase / Menstruation phase:
The cycle starts with bleeding phase in its first 3 to 5 days. During this bleeding phase the part
of the layer of endometrium gets shed off. Total loss of blood per day is about 20 ml, so an
average of 40 to 80 ml blood/cycle is lost. This blood can not clot due to presence of Fibrinolytic
enzymes.

(ii) Preovulatory/Proliferative phase / Follicular phase.

• After first four or five days this phase begins. During this phase, Due to release of GnRH,
Pituitary secretes FSH and LH to stimulate the ovarian follicle. The ovarian follicle now
begins to develop. Developing follicle now starts secreting an increasing amount of
oestrogen.

• The rising level of oestrogen causes the endometrium to proliferate and thicken. It also
causes increase in the vascularity and glandularity of the endometrium.

• Due to this, the hypothalamus releases more of GnRH. This GnRH induces the pituitary to
release more of FSH. The rising FSH levels now cause:

41
(i) further growth and development of ovarian follicle to form Graafian follicle
(ii) even further release of oestrogen from the theca interna of this developing follicle.

• As the oestrogen level goes on rising, by the end of 10 day the extreme levels of oestrogen
(which have by then caused maturation of Graafian follicle and growth of endometrium)
now give a positive feedback of high concentration of oestrogen causing a rise in GnRH and
LH secretion but due to release of inhibin by graffian follicle, FSH is not comparatively rised
therefore the LH secretion from the pituitary goes on rising. This abrupt rise (on 11th to 13th
day) in LH concentration in blood is called as LH surge.

• This LH now causes the Graafian follicle to rupture after partial completion of II meiotic
division in oocyte and thus the secondary oocyte (metaphase stage) released. The release
of egg (secondary oocyte) which occurs around 14 day is called as ovulation.

(iii) Post ovulatory/secretory phase/ Luteal Phase:


• Luteal phase last for 14 days. During this phase the level of Estrogen and progesterone will
rise while FSH and LH levels drop.
• If pregnancy does not occur after ovulation, then as the progesterone level rise, its rising
levels inhibits the release of GnRH from hypothalamus by negative feed back. Due to this
FSH, LH secretion by pituitary falls and thereby progesterone secretion by the corpus luteum
(which was due to influence of LH) also now falls.

• As the progesterone level drops, the corpus luteum begins to degenerate and transform in
corpus albicans (which can not secrete progesterone). Due to the lack of progesterone.

(i) The overgrown endometrium now begin to break and separate from the inner uterine
wall causing bleeding.

(ii) The uterine contraction (which was till now inhibited due to presence of progesterone)
now start. Thus the separated endometrium along with blood is now being passed out
via vaginal route. This is again the beginning of next menstrual or bleeding phase.

• The period between ovulation and next menstrual bleeding (post ovulatory period) is always
constant (i.e. 14 days). However, the ovulation date may vary (causing a change in pre
ovulatory period).

• In human beings, menstrual cycle ceases around 50 years of age, termed as menopause.
Cyclic menstruation is an indicator of normal Reproductive phase and extends between
menarche and menopause.
Menstrual Hygiene:
Maintenance of hygiene and sanitation during menstruation is very important. Take bath and clean
yourself regulatory. Use sanitary napkins or clean homemade pads. Change sanitary napkins or
homemade pads after every 4–5 hrs. as per the requirement. Dispose of the used sanitary napkins
properly wrapping it with a used sanitary napkins properly wrapping it with a used paper. Do not throw
the used napkins in the drainpipe of Toilets or in the open area. After handling the napkin wash hands
with soap.

42
Concept Builder

1. Withdrawal of which of the following hormones is the immediate cause of menstruation –


(1) Progesterone (2) Estrogen (3) FSH (4) FSH – RH

2. Repair of endometrium is undertaken by –


(1) LH (2) FSH (3) Estrogen (4) Prolactin

3. Luteal phase is the other name of:


(1) Follicular phase (2) Proliferative phase (3) Menstrual flow phase (4) Secretory phase

4. If the menstrual cycle is of 35 days then what is risk period (cycle start on 1st day):
(1) 9th to 17th days (2) 11th to 18th days (3) 16th to 24th days (4) 18th to 35th days

5. After ovulation graafian follicle converted into:


(1) Corpus luteum (2) Corrpus albicans (3) Corpus cavernosa (4) Corpus callosum

6. Events of menstrual cycle are the cyclic changes in the –


(1) Myometrium (2) Perimetrium (3) Endometrium (4) Epimetrium

7. In menstrual cycle on which day progestrone level rises –


(1) 1st to 5th day (2) 6th to 14th day (3) 13th to 14th day (4) 15th to 28th day

43
8. Period of the cycle is know as the proliferative phase is:-
(1) 1st to 5th day (2) 6th to 13th day (3) 15th to 28th day (4) 14th to 15th day

9. In what stage of menstural cycle does the corpus luteum form:-


(1) Proliferative phase (2) Follicular phase (3) Menses (4) Luteal phase

10. If pergnancy does not occur, the corpus luteum will disintegrate & turn into:-
(1) Corpus callosum (2) Corpus albicans (3) Corpus cavernosa (4) Corpus Epididymis

Concept Builder (Answer-Key)


Que. 1 2 3 4 5 6 7 8 9 10
Ans. 1 3 4 3 1 3 4 2 4 2
Fertilization

1. The process in which union of male and female gametes (formed by gametogenesis) and fusion of
pronuclei of sperm and ovum takes place thus diploid zygote is formed, is called fertilization.
Fertilization has following processes:- The union of male and female gametes is called Syngamy,
where as intermixing of their cytoplasm is called plasmogamy. The fusion of pronuclei of sperm
and ovum is called karyogamy. The intermingling of their chromosomes is called amphimixis.

Ovum surrounded by few sperms


2. During copulation (coitus) semen is released by the penis into the vagina of female, called
insemination. Prostaglandins of semen help in the movement of spermatozoa.
3. Mammalian sperms acquire activity at two places. First-epididymis and second-female
reproductive tract. Vaginal secretion make the sperm highly active and sperm acquire capacity of
fertilization is called capacitation.
4. Sperm swim through the vagina, cervix, uterus and finally reach the ampulla of the fallopian tubes.
The ovum released by the ovary is also transported to the ovum is released in the secondary oocyte
stage (arrested in metaphase-II). Due to ciliary current produced by fimbriae portion of oviduct,
ovum is drawn in through ostium. It reaches ampulla, the site of fertilization, by the ciliary action
of cilliated columnar epithelial lining of oviduct.

44
5. Fertilisation can only occur if the ovum and the sperms are transported simultaneously to the
ampulla. This is the reason why not all copulations lead to fertilisation and pregnancy.
6. A number of sperms adhere to the surface of egg (Agglutination). The acrosome starts releasing its
hydrolytic enzymes of sperms lysins which include.
(a) Hyaluronidase: Dissolves the hyaluronic acid responsible for cementing of follicle cells or
granulosa cells.
(b) Zona lysin /Acrosin: Digests the zona pellucida.
7. The entry of sperm into the ovum induces completion of the meiotic division of the secondary
oocyte. Entry of sperm causes breakdown of metaphase promoting factor (MPF) and turns on
anaphase promoting complex (APC). This results in completion of meiosis-II. The second meiotic
division is also unequal and results in the formation of a second polar body and a haploid ovum
(ootid). Soon the haploid nucleus of the sperm and that of ovum fuse together to form a diploid
zygote.
8. At the point of contact with sperm and plasma-membrane of egg a cone-like structure is formed
called reception cone. After some time reception cone sinks in egg cytoplasm along with sperm
(entry of sperm is a type of phagocytosis).
With the entry of sperm all the cortical granules burst and secrete a membrane around the egg is
called fertilization membrane (cortical reaction). It is secreted on inner surface of primary egg
membrane and perivitelline space become more wide and amount of perivitelline fluid is also
increase. Function of perivitelline fluid and fertilization membrane is to prevent the entry of sperm
in egg. so normally only one sperm enter inside the egg (monospermy). Sometimes more than one
sperm enter inside the egg (polyspermy).
9. Fate of sperm in egg:
• In majority of animals, only head and middle piece enter inside the egg and tail is left outside.
• In mammals, whole sperm enters in the egg.
• All the structures of sperm dissolve in egg cytoplasm except sperm nucleus and proximal
centriole.
• The centriole of egg itself degenerates at the time of second maturation division. So proximal
centriole of sperm starts division, it divides into 2 daughter centrioles, which migrate towards
opposite pole and start forming spindle fibers.

Fate of sperm nucleus:


• The nucleus of sperm absorbs water from egg cytoplasm and becomes enlarged. Now it is
called male pronucleus. Male pronucleus and Female pronucleus migrate through definite
routes and come close to each other. These routes are called fertilization path.

Cleavage (Cellulation or segmentation)


• The term 'Cleavage' was given by "Von Baer".
• In fertilized egg or activated egg, the egg undergoes repeated cell divisions which occur rapidly
producing a multicellular structure without changing its size. All these rapid mitotic cell
divisions are path collectively called cleavage or segmentation.

45
• Due to the process of cleavage, a single celled zygote, through a successive mitotic cell divisions
changes into a complex multicellular structure. Cells produced as a result of cleavage are
termed as blastomeres.
• The total size of the embryo remains the same. Though the number of blastomeres as a result
of mitotic cell divisions increases, the size of blastomeres gradually decreases as compared to
parent cell.
• Interphase stage is very short in cleavage. In interphase only DNA duplication and histone
protein synthesis takes place up to some extent.
In the interphase of cleavage only 'S' phase is present, G1 & G2 phases are absent.
Protein synthesis and RNA synthesis do not occur during this interphase.
• Size of blastomeres decreases during cleavage.
When size of blastomere becomes equal to that of size of somatic cells, the divisions of
cleavage are stopped.

Classification of Cleavage

On the basis of fate of blastomeres: -


(A) Determinate Cleavage - In this pattern of cleavage, the fate of blastomeres is fixed, determined
i.e. each blastomere forms a particular portion of embryo. If (by certain reason) any blastomere is
damaged or destroyed, then the part of embryo (which would have developed from that blastomere)
will be absent e.g., Nematoda, Annelida, Mollusca and . Some chordates like amphibia & ascidians.

(B) Indeterminate Cleavage - In this type of cleavage, the fate of blastomeres is not definite. All
the blastomeres form all the parts of embryo. If some blastomeres are lost, no loss is observed in
this embryo. If in the early stages of cleavage, the embryo is cut into small pieces, then each piece
of embryo will develop into a complete embryo, and all the embryos are identical. So identical
twins are monozygotic. This is the basis of embryo cloning.

General Stages of Embryonic Development


1. Morula - As a result of segmentation or cleavage activities, unicellular zygote changes into a solid
ball like multicellular structure. In the later stage of cleavage, clusters of sticky, cohering,
protruding (outside) blastomeres are produced, which look like mulberry. This 8-16 celled stage is
termed as morula stage.
• The mitotic division starts as the zygote moves through the isthmus of the oviduct called
cleavage towards the uterus and forms 2, 4, 8, 16, daughter cells called blastomeres. The
embryo with 8 to 16 blastomeres is called a morula.
2. Blastocyst Formation- Blastula of Eutherian & Metatherian mammals is called Blastocyst, because
blastula is in the form of a cyst.
• The blastomeres in the blastocyst are arranged into an outer layer called trophoblast and an
inner group of cells attached to trophoblast called the inner cell mass (embryonal knob). The
trophoblast layer then gets attached to the endometrium and the inner cell mass gets
differentiated as the embryo.
• Cell of trophoblast just above the embryonal knob are called cells of Rauber (amniogenic cells).
There occurs a cavity in between inner cell mass and trophoblast called albumin cavity
(Blastocoel). It is filled with nutritive fluid absorbed from the wall of uterus. So albumin cavity
is also nutritive- cavity.

46
• After attachment, the uterine cells divide rapidly and covers the blastocyst. As a result, the
blastocyst becomes embedded in the endometrium of the uterus. This is called implantation
and it leads to pregnancy.
• In human, the site of implantation is generally mid-dorsal/fundus part of uterus.
• Implantation of blastocyst takes about 7-8 days after fertilization in human and by 12th day it
is completely buried in the wall of the uterus. The place of entry through which the embryo
enters into the wall, is completely closed by a fibrous and cellular plug, known as closing
coagulum.
• After implantation, the wall of uterus is called as decidua, instead of endometrium.
Decidua also comes out from uterus at the time of parturition.
• Zona pellucida prevent implantation of the blastocyst at an abnormal site. The trophoblast has
the property of being able to stick to the uterine (or other) epithelium and its cells have the
capacity of eat up other cells.

Fig. Transport of ovum, fertilization and passage of growing embryo through fallopian tube
3. Gastrulation
Gastrula: Gastrula stage is the most important stage in embryonic development because two main
events take place during gastrula stage.
(a) Differentiation of blastomere: As a result of differentiation of blastomere; three germinal layers
i.e. ectoderm, mesoderm and endoderm are formed. Formation of three germinal layers is the
significance of gastrula stage.
(b) Morphogenetic Movements: During gastrula stage blastomere perform amoeboid movement and
reach to their definite place in embryo because after the gastrulation organogenesis has to start
in embryo.

Summary of developmental stages in human

• The first sign of growing foetus may be noticed by listening to the heart sound carefully through
the stethoscope. After one month of pregnancy, the embryo's heart is formed.
• By the end of the second month of pregnancy the foetus develops limbs and digits.
• By the end of 12 weeks (First trimester), most of the major organ system are formed.
• The first movements of the foetus and appearance of hair on the head are usually observed
during the fifth month.

47
• By the end of 24 weeks (Second trimester), the body is covered with fine hair, eye-lids separate
and eye lashes are formed.
• By the end of nine months of pregnancy, the foetus is fully developed and is ready for delivery.

Extra Embryonic Membranes and Placenta

Extra embryonic membranes


In chordates like reptiles, birds and prototherian mammals, blastula is a disc shaped structure called
as blastodisc. The cellular layer formed of blastomeres remains as blastoderm. The central part of
blastoderm gives rise to embryo proper, while the peripheral portion does not take part in the formation
of embryo. This peripheral part is known as extra embryonic region. This region takes part in the
formation of certain membranes called extra embryonic membranes. These extra embryonic
membranes provide facilities for nutrition, respiration and excretion to the embryo. Extra embryonic
membranes are of four types–
1. Amnion 2. Chorion 3. Yolk sac 4. Allantois

On the basis of presence or absence of amnion, two groups of vertebrates are categorised-
1. Amniota- This group is characterized with the presence of amnion in the embryos of its members.
For example members of class Reptilia, Aves and Mammalia.

2. Anamniota- Animals of this group are devoid of amnion in their embryos. For example class
cyclostomata, pisces and amphibia.

Fig. Formation of extraembryonic membranes in human


1. Amnion: With a gradual increase in size the amnion covers the embryo from all sides. After about
eight weeks of fertilization, amnion is completely incorporated into connecting stalk, which finally
forms the umbilical cord. Embryo, in this stage, is called as foetus remains hanging in amniotic
fluid.
2. Chorion: After implantation of blastocyst, the trophoblast gives out several finger like processes,
the chorionic villi which get embedded into uterine endometrium Mesoderm also contributes in the
formation of these villi. After a period of four month these villi disappear from all parts except the
connecting stalk where they grow rapidly and participate in the formation of placenta.
3. Yolk sac: Initially the size of yolk sac is larger as compared to that of the embryo. About eight
weeks after fertilization, the yolk is reduced in size and changes into a tubular structure. Ultimately
a placenta is developed with the incorporation of yolk sac and mesodermal connecting stalk with
the amnion and chorion.

48
4. Allantois: The mesoderm of allantois forms many small blood vessels in this region. These vessels
connect the embryo with placenta and ensure nutritional and respiratory supply to embryo. In
human, allantois does not function to store the excretory wastes as it does in reptiles and birds.

Placenta
• After implantation, finger-like projections appear on the trophoblast called chorionic villi which are
surrounded by the uterine tissue and maternal blood. The chorionic villi and uterine tissue become
interdigitated with each other and jointly form a structural and functional unit between developing
embryo (foetus) and maternal body called placenta.
• Placenta is found in all viviparous animals. Exept sub-class-prototheria;
• The placenta facilitate the supply of oxygen and nutrients to the embryo and also removal of carbon
dioxide and excretory/waste materials produced by the embryo.
• The placenta is connected to the embryo through an umbilical cord which helps in the transport of
substances to and from the embryo.
• Placenta also acts as an endocrine tissue and produces several hormones like human chorionic
gonadotropin (hCG), human placental lactogen (hPL), estrogens, progestogens, etc. In the later
phase of pregnancy, a hormone called relaxin is also secreted by the placenta.
• HCG, HPL and relaxin are produced in women only during pregnancy.
• In addition, during pregnancy the levels of other hormones like estrogens, progestogens, cortisol,
prolactin, thyroxine, etc., are increased severalfolds in the maternal blood. Increased production of
these hormones is essential for supporting the fetal growth, metabolic changes in the mother and
maintenance of pregnancy.

Fig. The human foetus within the uterus


Parturition

• The average duration of human pregnancy is about 9 months which is called the gestation period.
• Vigorous contraction of the uterus at the end of pregnancy causes expulsion/delivery of the foetus.
This process of delivery of the foetus (childbirth) is called parturition.
• Parturition is induced by a complex neuroendocrine mechanism.
• The signals for parturition originate from the fully developed foetus and the placenta which induce
mild uterine contractions called foetal ejection reflex.
• This triggers release of oxytocin from the maternal pituitary. Oxytocin acts on the uterine muscle
and causes stronger uterine contractions, which in turn stimulates further secretion of oxytocin.
The stimulatory reflex between the uterine contraction and oxytocin secretion continues resulting
in stronger and stronger contractions. The labour pain during child birth, is due to this hormone.
Oxytocin is the main parturition hormone. Oxytocin stimulates milk let down by milk ejection reflex.

49
• Relaxin hormone is secreted by the placenta and the ovary of pregnant female. This hormone
relaxes the public symphysis i.e. the joint between the pelvic girdles. So more space is available to
the foetus to move out.
• Soon after the infant is delivered, the placenta is also expelled out of the uterus.

Lactation
• The mammary glands of the female undergo differentiation during pregnancy and starts producing
milk towards the end of pregnancy by the process called lactation. This helps the mother in feeding
the newborn.
• The milk produced during the initial few days of lactation is called colostrum which contains several
antibodies absolutely essential to develop resistance for the new-born babies. Breast-feeding
during the initial period of infant growth is recommended by doctors for bringing up a healthy baby.
Special Points
Gynaecomastia – Development of breast in the male.
Amenorrhoea – Absence of menstruation cycle.
Hysterectomy – Surgical removal of uterus.
Oopherectomy – Removal of ovaries.

Concept Builder

1. The average duration of human pregnancy is about –


(1) 10 weeks (2) 28 weeks (3) 32 weeks (4) 9 months

2. Mammary glands of the female undergo differentiation during pregnancy & starts producing milk –
(1) At the end of pregnancy (2) At the end of 1st trimester
(3) During pregnancy (4) Before pregnancy
3. Breast feeding during initial period is –
(1) Good for healthy baby
(2) Recommended by doctors
(3) Essential to develop resistance to the new born babies
(4) All of these
4. The role of placenta is –
(1) To convey nerve impulses (2) To act as storage organ
(3) To protect mother from shocks (4) To provide nutrition for developing embryo
5. Placenta produce which hormone
(1) ACTH (2) Progesterone (3) GH (4) Gastrin
6. Morphogenetic movements take palce in –
(1) Morula (2) Blastula (3) Gastrula (4) Organogenesis
7. Amniotes includes -
(1) Fishes, reptiles & amphibians (2) Amphibians, birds & reptiles
(3) Birds, reptiles & mammals (4) Amphibians, reptiles & mammals

Concept Builder (Answer-Key)


Que. 1 2 3 4 5 6 7
Ans. 4 1 4 4 2 3 3

50
Exercise - I
1. Primary sex organ differ from the 6. The primary regulator of Leydig cell
secondary sex organs in all the following secretion is:
except: (1) FSH releasing factor
(1) They produce gametes (2) Androgen-binding protein
(3) Luteinizing hormone (LH)
(2) They secrete sex hormones
(4) Follicle stimulating hormone
(3) They are concerned with the
conduction of gametes 7. Testosterone is secreted by:
(4) Testes in male and ovaries in female (1) Leydigs cells (2) Sertoli cells
are the examples of primary sex (3) Pituitary gland (4) Testis
organs
8. Outer coat of seminiferous tubules is
composed of fibrous connective tissue
2. The organ which produces gametes are
called:
called ..A.. and which neither produces
(1) Tunica propria (2) Lamina propria
gametes nor hormones are called ..B.. .
(3) Plica semilunaris (4) Tunica albuginea
Here A and B represent.
(1) A – primary sex organs; 9. At the time of sexual excitation, crura
B – Secondary Sex organs muscles in penis:
(1) Relaxed (2) Contracted
(2) A – Secondary sex organs;
(3) Collapsed (4) None
B – Primary Sex organs
(3) A – Tertiary sex organs; 10. Scrotal sacs of man are connected with
B – Secondary Sex organs the abdominal cavity by:
(1) Inguinal canal (2) Haversian canal
(4) A – Secondary sex organs;
(3) Vagina cavity (4) Spermatic canal
B – Tertiary Sex organs
11. Partitions of testis develop from: -
3. Gubernaculum is the ligamentous (1) tunica vasculosa
connective cord which connects: (2) tunica albuginea
(1) Testis to kidney (3) tunica vaginalis
(2) Testis to scrotum (4) rete testis

(3) Ovary to abdominal wall 12. Which is not a secondary sex organ ?
(4) Testis to Abdominal cavity (1) Vagina (2) Penis
(3) Prostate gland (4) Mammary gland
4. The tunica albuginea is a covering
around the: 13. If testes of a male are not transferred
from abdominal cavity to scrotal sac
(1) Testes (2) Kidneys
then:
(3) Uterus (4) Epididymis
(1) Person dies
(2) Absence of male characters
5. Testosterone is a/an:
(3) Development of male reproductive
(1) Steroid (2) Protein
system will not occur
(3) Octapeptide (4) Glycoprotein (4) Sperms will not form

51
14. Penile urethra traverses through: 22. Which one is unpaired gland in male
(1) Corpora cavernosa reproductive system ?
(2) Corpus spongiosum (1) Seminal vesicle (2) Cowper's gland
(3) Prostate gland (4) Lacrimal gland
(3) Corpus callosum
(4) Corpus striatum 23. Sugar fructose is present in the
secretion of:
15. Cryptorchidism is a condition of testes: (1) Seminal vesicle (2) Perineal gland
(1) Unable to descend in scrotal sacs (3) Cowper's gland (4) Bartholin's gland
(2) Unable to produce sperms
24. What would happen if vas deferens of
(3) Having been surgically removed
man are cut?
(4) Having remained undeveloped (1) Semen is not formed
(2) Spermatogenesis does not occur
16. Sertoli cells are found in:
(3) Semen is without sperms
(1) Testis of mammal (4) Sperm are non motile
(2) Ovary of mammal
(3) Testis of Ascaris 25. Leydig cells are found in:
(1) Seminiferous tubules
(4) Pancrease of frog
(2) Testis
17. Which of the following controls the (3) Ovary
function of Sertoli cells ? (4) Epididymis

(1) FSH (2) ICSH 26. Temperature in scrotum necessary for


(3) Oestrogen (4) Testosterone sperm formation should be:
(1) 2oC above than body temperature
18. Cauda epididymis leads to: (2) 2oC below than body temperature
(1) Rete testis (2) Vas efferens (3) 8oC above than body temperature
(3) Vas deferens (4) Ejaculatory duct (4) 8oCbelow than body temperature

19. Major part of semen is secreted by: 27. Accessory glands of male reproductive
system are:
(1) Seminal vesicle (2) Prostate gland
(1) Prostate and seminal vesicles
(3) Cowper's gland (4) Bartholin's gland
(2) Prostate, Bartholin’s and seminal
vesicles
20. In male, the acidity in the urethra is
(3) Seminal vesicles and Bartholin’s
neutralized by the secretions of:
(4) Prostate, Cowper’s and seminal
(1) Cowper's gland (2) Bartholin glands
vesicles
(3) Perineal glands (4) Leydig cells
28. Epididymis is:
21. Seminal plasma contains the secretions of: (1) Network of sinuses between
(1) Follicles, uterus and prostate gland seminiferous tubules and vasa
(2) Prostate, Cowper's and Bartholin's efferentia
(2) Intermidiate structure between rete
gland
testis and vasa effferentia
(3) Seminal vesicle, uterus and prostate
(3) A long coiled tube between vasa
gland efferentia and vas deferens
(4) Seminal vesicle, prostate and (4) Connection between vas deferens
Cowper's gland and seminal vesicle

52
29. Which of the following releases inhibin Answer codes:
to control spermatogenesis ? (1) A = 1, B = 2, C = 3, D = 5, E = 4
(1) Rete testis (2) Follicular cells (2) A = 3, B = 1, C = 4, D = 2, E = 5
(3) Leydig's cells (4) Sertoli cells (3) A = 2, B = 4, C = 3, D = 5, E = 1
(4) A = 3, B = 1, C = 2, D = 5, E = 4
30. Spermatogenesis and sperm
36. Correct sequence of different layers of
differentiation are under the control of:
uterine wall is (respectively from outside
(1) FSH to inside) :-
(2) TSH (1) Perimetrium → Endometrium →
(3) Progesterone Myometrium
(4) Parathyroid Harmone (2) Myometrium → Perimetrium →
Endometrium
31. Testes descend into scrotum in
(3) Endometrium → Myometrium →
mammals for: Endometrium
(1) Spermatogenesis (4) Perimetrium → Myometrium →
(2) Fertilization Endometrium
(3) Development of sex organs
37. Penis is male external genitalia which
(4) Development of visceral organs
has three cylindrical masses of erectile
32. Which cells are found in between tissue. The erect penis shows:
spermatogonia? (1) Two dorsal corpus spongiosum and
(1) Germinal cells (2) Epithelial cells one ventral corpora cavernosa
(2) Two dorsal corpora cavernosa and
(3) Sertoli cells (4) Lymphatic space
one ventral corpus spongiosum
33. Secondary sex organ is: (3) Two ventral corpora cavernosa and
(1) Testis (2) Ovary one dorsal corpus spongiosum
(3) Beard (4) Vasa deferens (4) One dorsal corpora cavernosa and
two ventral corpus spongiosum
34. Formation of sperms occurs in: -
38. Abdominal ostium is the aperture
(1) rete testis present in: -
(2) seminiferous tubules (1) oviduct
(3) Both (1) & (2) (2) fimbriated fallopian funnel
(4) Mediastinum testis (3) ovary
(4) cloaca
35. Match the following:
39. Which is not correct about sertoli cells ?
Set I Set II
(1) It is situated in between the
A. Inguinal 1. Network of channels germinal epithelial cell
canal after seminiferous (2) It is related with the nutrition of
tubules sperm
B. Rete 2. Androgen (3) It forms blood testis barrier
(4) It secretes testosterone
testis
C. Leydig 3. For descending of 40. Bartholin’s glands occur in: –
(1) Females and produce oestrogen for
cells testis
regulating secondary sexual
D. Prepuce 4. Dorsal bundles of characters
spongy tissues (2) Males and form liquid part of semen
E. Corpora 5. Terminal skin of penis (3) Females and help in vestibular lubrication
(4) Males and produce alkaline fluid for
cavernosa
neutralising urethral acidity.

53
41. Synthesis of testosterone by Leydig cells 50. Identify the odd one from the following:
is stimulated by: (1) Labia minora (2) Fimbriae
(1) GH (2) TSH (3) FSH (4) ICSH (3) Infundibulum (4) Isthmus

42. Function of prostate glands is: 51. Endometrium is lining of:


(1) Storage of semen (1) Testis (2) Urinary bladder
(2) Provide motility to sperms (3) Uterus (4) Ureter
(3) Formation of sperm
52. Bartholin's glands occurs in:
(4) Release of hormones
(1) Females and help in vestibular
43. Which one of the following is not a male lubrication
accessory gland ? (2) Females and produce oestrogen for
(1) Seminal vesicle regulating secondary sexual
characters
(2) Ampulla
(3) Prostate (3) Males and form liquid part of
spermatic fluid
(4) Bulbourethral gland
(4) Males and produce alkaline fluid for
44. The head of the epididymis at the head neutralising urethral acidity.
of the testis is called:
53. Which of the following is not related to
(1) Cauda epdidymis (2) Vas deferense
vulva ?
(3) Caput epididymis (4) Gubernaculum
(1) Mons-pubis (2) Clitoris
45. If the epididymis is being removed, then (3) Labia majora (4) Cervix
what will happen ?
54. The wall of the uterus has three layers
(1) Short life span of sperm
of tissue. The layer which undergoes
(2) Early cross the pathway
cyclical change during menstrual cycle
(3) Sperm will be incapable for
is:
fertilization
(1) Perimentrium (2) Myometrium
(4) Functional maturation is early
(3) Endometrium (4) Both (2) & (3)
46. When do both LH & LSH attain a peak 55. Labium majora of a female mammal is
level in a menstrual cycle. homologous to:
(1) In last week of the cycle (1) Scrotal sac (2) Prostate gland
(2) In mid of the cycle (3) Epididymis (4) Seminal vesicle
(3) During Initial days of cycle
(4) On 4th day of cycle 56. ‘Spermiogenesis’ is a process in which: -
(1) spermatocytes give rise to
47. Development of foetus takes place in: spermatozoa
(1) Vagina (2) Uterus (2) spermatogonium produces a
(3) Ovary (4) Oviduct spermatid
(3) Spermatids are changed into
48. The mitotic division start as the zygote
spermatozoa
moves through the ______ of the oviduct
(4) dormant spermatozoa become
called cleavage towards the uterus.
active just before ejaculation
(1) Isthmus (2) Ampulla
(3) Fimbriae (4) Infundibulum 57. Abnormal conditioning when the
mammary glands of man become female
49. Lower narrow end of uterus is called: like is called:
(1) Urethra (2) Cervix (1) Feminization (2) Gonochorism
(3) Fundus (4) Vulva (3) Gynacomastism (4) Gynoecism

54
58. Expanded proximal part of oviduct in 66. First menstruation begins at puberty
female is: and is called:
(1) Uterus (2) Ampulla (1) Menses (2) Menopause
(3) Isthmus (4) Infundibulum (3) Menarche (4) Implantation

59. When both ovary are removed from


67. In Human, duration of menstrual cycle is:
human then which hormone is
decreases in blood ? (1) 21 days (2) 28 days
(1) Oxytocin (3) 38 days (4) 40 days
(2) Prolactin
68. Stages in menstrual cycle are:
(3) Estrogen
(1) Recovery and proliferative phase
(4) Gonadotrophic releasing factor
(2) Proliferative and secretory phase
60. Correctly matched pairs are: (3) Proliferative, secretory and
1. Clitoris - Erectile body in female menstrual phase
homologous to penis of male
(4) Recovery phase, secretory phase and
2. Sexual intercourse - coitus
phase of menstrual flow
3. Colostrum - Secretion found in
seminal fluid 69. Cessation of menstrual cycle is called:
4. Areola - Pigmented circular area
(1) Ovulation (2) Menopause
around the nipple.
(3) Menarche (4) Menses
Answer codes:
(1) 1 and 2 are correct 70. Phase of menstrual cycle in human that
(2) 2 and 4 are correct lasts for 7-8 days is:
(3) 1 and 3 are correct
(1) Follicular phase (2) Ovulatory phase
(4) 1, 2 and 4 are correct
(3) Luteal phase (4) Menstruation
61. Womb is the another name of:
71. Stem cells are found in: -
(1) Vagina (2) Cervix
(3) Oviduct (4) Uterus (1) Ectoderm
(2) Endoderm
62. Identify the structure belongs to female (3) Inner cell mass
external genitalia:
(4) Mesoderm
(1) labia minora (2) Fimbriae
(3) Infundibulum (4) Isthmus 72. Progesterone level falls leading to:

63. Which temporary endocrine gland forms (1) Gestation (2) Menopause
in ovary after ovulation ? (3) Lactation (4) Mensturation
(1) Corpus callosum (2) Corpus albicans
(3) Corpus luteum (4) Corpus striata 73. If menstual cycle is 30 days & bleeding
start on Ist day then ovulation occur on:
64. Corpus luteum secretes: (1) 14th day (2) 18th day
(1) LH (2) Oxytocin (3) 30th day (4) 16th day
(3) Progesterone (4) FSH
74. Pregnancy hormone is:
65. Corpus luteum is:
(1) Estrogen (2) Progesterone
(1) Excretory (2) Endocrine
(3) Oxytocin (4) FSH
(3) Digestive (4) Reproductive

55
75. Some important events in the human 79. The following graph of relative
female reproductive cycle are given concentration of the four hormones
below. Arrange the events in a proper present in the blood plasma of a woman
sequence: during her mestural cycle. Identify the
A - Secretion of FSH, hormones. A, B, C and D.
B - Growth of corpus luteum,
C - Growth of the follicle and oogenesis,
D - Ovulation,
E - Sudden increase in the levels of LH
(1) ADCEB (2) BACDE
(3) ACEDB (4) CADBE

76. Which one of the following statements (1) A– FSH, B– Progesterone, C– LH,
is incorrect about menstrual cycle ? D – Oestrogen
(1) The first menstruation begins at the (2) A– LH, B– Progesterone, C– FSH,
puberty and is called menarche. D – Oestrogen
(2) Lack of menstruation may also occur (3) A– FSH, B– Oestrogen, C– LH,
due to some factors like stress, poor D – Progesterone
health. (4) A– LH, B– Oestrogen, C– FSH,
(3) Corpus luteum secretes large D – Progesterone
amounts of progesterone which is 80. Match the hormones in column I with
essential for maintenance of their functions in column II. Choose the
endometrium answer, which given the correct
(4) In absence of fertilisation, corpus combination of the two columns.
luteum degenerates in luteal phase Column I Column II
and new folicles starts developing Perpare endometrium wall for
immediately due to progesterone. A. FSH 1.
implantation
Develops female secondary
77. Ovulation in the human female normally B. LH 2.
sexual characters
takes place during the menstrual cycle: C. Progesterone 3. Contraction of uterine wall
(1) At the end of the proliferative phase D. Oestrogen 4. Development of corpus luteum
(2) At the mid secretory phase 5. Maturation of Graafian follicle
(3) Just before the end of the A B C D
seecretory phase (1) 5 4 1 2
(4) At the beginning off the proliferative (2) 4 5 2 1
phase (3) 4 3 2 5
(4) 5 1 2 4
78. The cause of sudden increase of the LH
hormone in the middle of the cycle 81. Which of the following hormone initiates
(about 14th day) is: a metabolic rise that results into the
(1) Negative feedfack of progesterone rupture of graafian follicle?
on the hypothalamus (1) Prolactin (2) HCG
(2) Negative feedback of estrogen on (3) FSH (4) LH
the anterior lobe of pituitary
(3) Positive feedback of FSH on the 82. Level of estrogen and progesterone are
overy minimum at the time of:
(4) Positive feedback of estrogen on the (1) Follicular phase (2) Ovulation
anterior lobe of pituitary (3) Secretory phase (4) Menses

56
83. What is the oligospermia condition? 89. Acrosome formation in spermatogenesis
(1) If sperm count is 40 to 120 milion/ml occurs in which stage?
(2) If sperm count is < 20 million/ml (1) spermiogenesis
(3) If sperm count is < 60 million/ml (2) First meiotic division
(4) If sperm count is 20 million to 40 (3) growth phase
million/ml (4) spermiogenesis
84. For normal fertility:
90. How many polar bodies are produced
(1) Atleast 60% sperm must show
during the entire process of oogenesis in
vigrous motility
unmarried/virgin human female?
(2) Atleast 40% sperm must have
normal shape and size (1) Three (2) Two

(3) Atleast 30% sperm must have (3) Four (4) One
normal activity and function
91. During spermatogenesis how many
(4) Atleast 40% sperm must show
sperms are formed from a single primary
vigrous motility or 60% sperm must
spermatocyte:
have normal shape and size
(1) 1 (2) 2 (3) 4 (4) 8
85. Which piece of a sperm is called power
house ? 92. How many secondary spermatocytes
(1) Head piece (2) Neck piece will form 400 spermatozoa?
(3) Middle piece (4) Tail piece (1) 100 (2) 400 (3) 40 (4) 200

86. "Spermiogenesis" is a process in which: 93. Which hormone acts on uterine


(1) Spermatids change into myometrium during parturition?
spermatozoa (1) LH
(2) Spermatogonia produce a spermatid
(2) Estrogen
(3) Spermatocytes give rise to
(3) Relaxin
spermatozoa
(4) Oxytocin
(4) Dormant spermatozoa become
active just before ejaculation. 94. The head of a mature sperm is mainly
87. In gametogenesis, reduction division composed of:
take place during: (1) Elongated nucleus and acrosomal
(1) Multiplication phase material
(2) Growth phase (2) Mitochondria, cytoplasm & nucleus
(3) First maturation division (3) Two centriole & the axial filament
(4) Second maturation (4) All of the above

88. Correct order of spermatogenesis is: 95. Cytoplasm of ovum does not contain:
(1) Primary Spermatocytes → Spermatogonia (1) Ribosomes (2) Mitochondria
→ Spermatid → Sperm (3) Goldi bodies (4) Centrosomes
(2) Spermatogonia → Spermatid →
Spermatocytes → Sperm 96. First meiotic division during Oogenesis
(3) Spermatid → Spermatogonia → occurs in:
Spermatocytes → Sperm (1) Oogonia
(4) Spermatogonia → Primary Spermatocytes (2) Second polar body
→ Secondary Spermatocytes → (3) Primary oocytes
Spermatid → Sperm (4) Secondary oocytes

57
97. The minute cells which separate from 103. The process of spermatogenesis and
the developing ova during their oogenesis in most vertebrates are under
maturation are called the influence of which hormone:
(1) Primary Oogonia (1) Oxytocin (2) FSH
(3) ACTH (4) LH
(2) Secondary Oogonia
(3) Polar bodies 104. Number of eggs released in the life time
(4) Primary spermatogonia of a woman is approximately :
(1) 4,00,000 (2) 450
98. Which of the following is haploid ? (3) 4000 (4) 1,60,000
(1) Primary spermatocytes & primary
105. A glycoprotein non-cellular membrane
Oocytes
which normally surrounds the ovum of a
(2) Secondary spermatocytes & secondary mammal:
Oocytes (1) Corona radiata
(3) Spermatogonia and Oogonia (2) Jelly envelope
(4) Spermatogonia and secondary (3) Zona pellucida
oocyte (4) Granulosa membrane

99. How many sperm and ova will be formed 106. Which one holds corona radiata cells
from 50 secondary oocytes and 50 together ?
secondary spermatocytes ? (1) Lipoprotein
(1) 50 ova & 200 sperm (2) Liposaccharide
(3) Oligosaccharide
(2) 50 ova & 100 sperm
(4) Mucopolysaccharide
(3) 100 ova & 200 sperm
(4) 100 ova & 400 sperm
107. Which is the correct sequence of layers
in the mammalian egg from outside to
100. Polar body is produced during the
inside?
formation of:
(1) Zona pellucida, corona radiata,
(1) Sperm (2) Secondary oocyte plasma membrane
(3) Oogonium (4) Spermatocytes (2) Corona radiata, zona pellucida,
plasma membrane
101. A human female has the maximum (3) Plasma membrane, zona pellucida,
number of primary oocytes in her corona radiata
ovaries: (4) Corona radiata, Plasma membrane,
(1) At menopause Zona pellucida,
(2) At Puberty
108. Oocyte is liberated from ovary under the
(3) At Birth influence of LH, after completing:
(4) Early in her fertile years (1) Meiosis I and after liberating second
polar bodies
102. Eggs librated from ovary in human is: (2) Meiosis I and before liberating
(1) Secondary oocyte stage second polar bodies
(2) Primary oocyte stage (3) Meiosis II and liberating second polar
(3) Oogonial stage bodies
(4) Mature ovum stage (4) Meiosis II after release of first polar
body

58
109. Stage of embryo development at which 116. Site of fertilization in mammal is:
implantation occurs in human female is: (1) Ovary (2) Uterus
(1) Morula (2) Zygote (3) Vagina (4) Fallopian tube
(3) Blastocyst (4) Gastrula 117. Acrosome reaction in sperm is triggeres:
(1) Capaciation
110. Identify the stage of sperm formation
(2) Release of zona lysin
during which the cytoplasmic volume of
(3) Influx of Na+
spermatid reduces:
(4) Release of fertilizin.
(1) Spermiogenesis
(2) Spermatidogenesis 118. Part of sperm involved in penetrating
egg membrane is:
(3) Spermatocytogenesis
(1) Tail (2) Acrosome
(4) Spermiation
(3) Middle Piece (4) Centriole
111. Secondary egg membrane is formed by: -
119. If Cowper’s gland is removed, then
(1) Oocyte (2) uterus which of the following would be
(3) oviduct (4) ovary affected?
(1) Sexual attraction
112. At what stage of life is oogenesis (2) Capacitation of sperms
initiated in a human female ?
(3) Copulation and fertilization
(1) At puberty (4) Hardness of penis
(2) During menarch
120. Which one of the following is incorrect ?
(3) During menopause
(1) Fertilization follows capacitation
(4) During embryonic development
(2) Cleavage of fertilized ovum results in
113. The acrosome plays a role in: blastula
(3) Fusion of sperm and ovum occurs in
(1) Fussion of nuclei of gametes
fallopian tube
(2) Motality of sperm
(4) Cleavage leads to increase in the
(3) Penetration of sperm in to ovum mass of protoplasm
(4) All of the above
121. Which of the following enzyme helps
114. The change in a mammalian sperm sperm to penetrate zona pellucida ?
which prepares it to fertilize the ovum is (1) Hyaluronidase
termed: (2) Neuraminidase
(1) Maturation (2) Preparation (3) Acrosin/zonalysin
(3) Capacitation (4) etamorphosis (4) Corona penetrating enzyme

115. Fertilization is: 122. Why do all copulations not lead to


fertilisation and pregnancy ? The root
(1) Union of diploid spermatozoa with
cause is .......
diploid ovum to form diploid zygote
(1) Due to numerous sperms and one
(2) Union of haploid sperm with haploid
ovum
ovum to form haploid zygote
(2) Due to less progesterone
(3) Union of haploid sperm with haploid
(3) Ovum and sperms are not
ovum to form diploid zygote transported simultaneously to the
(4) Union of diploid sperm with haploid ampullary region.
ovum to form triploid zygote (4) Due to non-formation of corpus luteum

59
123. Intermixing of cytoplasm of sperm and 131. The first movements of the foetus and
egg is known as: – apperance of hair on the head are
(1) Syngamy (2) Karyogamy usually observed during the:
(3) Amphimixis (4) Plasmogamy (1) 3rd month (2) 4th month
(3) 5th month (4) 8th month
124. Clevage starts in zygote:
(1) In Uterus 132. Archenteron is cavity in:
(2) In Fallopian tube (1) Blastula (2) Gastrula
(3) In Vagina (3) Zygote (4) Morula
(4) Cervix
133. Blastopore is the pore of:
125. Which of the following releases inhibin (1) Archenteron (2) Blastocoel
to control spermatogenesis?
(3) Coelom (4) Albumin cavity
(1) Rete testis
(2) Sustentacular cells 134. Morphogenetic movements can be seen
(3) Leydig’s cells (4) Follicular cells in:
(1) Marula (2) Blastula
126. Which of the following characteristics
(3) Gastrula (4) Placenta
does not belong to cleavage ?
(1) Decrease in size of blastomeres. 135. Gastrulation is a process of:
(2) Rapid mitotic cell division. (1) Formation of archenteron
(3) Interphase of very short duration. (2) Migration of prospective
(4) Differention of blastomeres. endomesodermal cells
(3) Differentiation of three primary germ
127. Which hormone is secreted in women
layers
only during the pregnancy?
(4) All of the above
(1) Progesterone (2) hPL
(3) Estrogen (4) Thyroxin 136. Correct sequence in development is:
(1) Fertilization → Zygote → Cleavage
128. In male, penis is covered by a loose fold
→ Morula → Blastula → Gastrula
of skin called as: -
(2) Fertilization → Zygote → Blastula
(1) Foreskin
→ Morula → Cleavage → Gastrula
(2) Urethral meatus
(3) Fertilization → Cleavage → Morula
(3) External genitalia
→ Zygote → Blastula → Gastrula
(4) Fimbriae
(4) Cleavage → Zygote → Fertilization
129. What is true for cleavage ? → Morula → Blastula → Gastrula
(1) Size of embryo increases 137. Stage of embryonic development in
(2) Size of Blastomeres decrease which differentiation of cell occurs:
(3) Size of Blastomeres increase (1) Blastula (2) Morula
(4) Size of embryo decreases (3) Gastrula (4) Neurula

130. Three germ layers are formed during 138. Solid ball of cell produced by repeated
which stage of Embryonic development: cleavage is called:
(1) Morula (2) Blastrula
(1) Gastrula (2) Blastula
(3) Gastrula (4) In any two stages
(3) Morula (4) Neurula

60
139. Which of the following statements is 147. The 16 cells stage of the human embryo is:
wrong? (1) Smaller than the fertilized egg
(1) Mammary lobes contain clusters
(2) Same size as the fertilized egg
called as alveoli.
(2) The last part of the oviduct is called (3) Two times of the size of the fertilized egg
as ampulla. (4) Four times the size of the fertilized egg
(3) Stroma of ovary is divided into two
zone 148. Match the following and choose the
(4) Uterus is also called as womb. correct
options:
140. In which stage of development the
embryonic cells form the germinal layers Set-I Set-II
by the movement: Embedding of
(1) Morula (2) Blastula A. Trophoblast (i) blastocyst in the
(3) Gastrula (4) Nerula endometrium
Group of the cells that
141. In which phase of menstrual cycle B. Cleavage (ii) would differentiated as
Graafian follicle is transformed into
embryo
corpus luteum?
(1) Proliferative phase Inner cell Outer layer of blastocyst
C. (iii)
(2) Luteal phase mass attached to the endometerium
(3) Growth phase
(4) Follicular phase D. Implantation (iv) Mitotic division of zygote

142. In human embryo the extra embryonic (1) A-ii, B-i, C-iii, D-iv
membrane are formed by: (2) A-iii, B-iv, C-ii, D-i
(1) Inner cell mass (2) Trophoblast (3) A-iii, B-iv, C-i, D-ii
(3) Formative cells (4) Follicles cells (4) A-ii, B-iv, C-iii, D-i
143. During preganancy, the urine of female
would contain: 149. After one month of pregrancy, the
(1) LH (2) Progesterone embryo’s ..A.. is formed. By the end of
(3) FSH (4) HCG the ..B.. month of pregnancy , the foetus
develops limbs and digits. By the end of
144. During fertilization, a sperm comes in
..C.. most of the major organ systems
contact with which layer of the ovum?
(1) Jelly coat are formed for example, the limbs and
(2) Vitelline membrane external genital organs are well-
(3) Perivitelline space developed. By the end of ..D.. the body
(4) Zona pellucida is covered with fine hair, eyelids
separate, and eyelashes are formed.
145. Which germ layer develops first during Here A to D refers to
embryonic development ?
(1) A–heart, B–second, C–first
(1) Ectoderm (2) Mesoderm
trimester, D–second trimester
(3) Endoderm (4) Both (2) and (3)
(2) A–heart, B–second, C–first month,
146. The fertilized egg in human female is D–second month
implanted in the uterus after:
(3) A–heart, B–second, C–first week,
(1) One month of fertilization
D– second week
(2) Two months of fertilization
(3) Three weeks of fertilization (4) A–heart, B–fourth, C–first trimester,
(4) About seven days of fertilization D–second trimester

61
150. Find out the incorrect match w.r.t. 153. In parturition process, which of the
development in humans: following does not happen:
(1) By the end of 24 weeks – Eye lids (1) Oxytocin Hormone is secreted by
separate and eyelashes are formed posterior pituitory
(2) By the end of 8 weeks – Appearance (2) Relaxin hormone responsible for
of hair on head and first movement narrowing of pelvic cavity
of foetus (3) Progesterone hormone secretion is
(3) By the end of 12 weeks – Limbs and stopped
external genital organs are well (4) Vigorous contractions of the uterus
developed
(4) By the end of 4 weeks – Heart is 154. The expulsion of completely developed
formed foetus from the uterus is known as:
(1) Ovulation (2) Oviposition
151. Placenta is the region where: (3) Gestation (4) Parturition
(1) Foetus is attached to mother by
spermatic cord
155. Which of the following placental
(2) Foetus is provided with mother's
hormone takes over the function of LH
blood
and maintains corpus luteum of
(3) Foetus receives nourishment from
pregnancy ?
mother's blood
(1) Human chorionic somatomammotropin
(4) Foetus is covered by membranes.
(2) Human chorionic corticotropin
152. Villi of human placenta develop from: (3) Human chorionic thyrotropin
(1) Chorion (2) Allantois (4) Human chorionic gonadotropin
(3) Yolk sac (4) Amnion

ANSWER KEY
Que. 1 2 3 4 5 6 7 8 9 10 11 12 13 14 15 16 17 18 19 20 21 22 23 24 25
Ans. 3 1 2 1 1 3 1 1 1 1 2 4 4 2 1 1 1 3 1 1 4 3 1 3 2
Que. 26 27 28 29 30 31 32 33 34 35 36 37 38 39 40 41 42 43 44 45 46 47 48 49 50
Ans. 2 4 3 4 1 1 3 4 2 4 4 2 2 4 3 4 2 2 3 3 2 2 1 2 1
Que. 51 52 53 54 55 56 57 58 59 60 61 62 63 64 65 66 67 68 69 70 71 72 73 74 75
Ans. 3 1 4 3 1 3 3 4 3 4 4 1 3 3 2 3 2 3 2 1 3 4 4 2 3
Que. 76 77 78 79 80 81 82 83 84 85 86 87 88 89 90 91 92 93 94 95 96 97 98 99 100
Ans. 4 1 4 3 1 4 4 2 4 3 1 3 4 1 4 3 4 4 1 4 3 3 2 2 2
Que. 101 102 103 104 105 106 107 108 109 110 111 112 113 114 115 116 117 118 119 120 121 122 123 124 125
Ans. 3 1 2 2 3 4 2 2 3 1 4 4 3 3 3 4 2 2 3 4 3 3 4 2 2
Que. 126 127 128 129 130 131 132 133 134 135 136 137 138 139 140 141 142 143 144 145 146 147 148 149 150
Ans. 4 2 1 2 3 3 2 1 3 4 1 3 3 2 3 2 2 4 4 3 4 2 2 1 2
Que. 151 152 153 154 155
Ans. 3 1 2 4 4

62
Exercise - II

1. Fructose is present in the secretion of: 8. Read the following statement carefully
and choose the incorrect statements:
(1) Corpus spongiosum
(1) The secretions of prostate glands
(2) Seminal vesicles also helps in the lubrication of the
(3) Bartholin gland penis.
(2) The stroma of ovary divided into two
(4) Prostate gland
zones – a peripheral cortex and an
inner medulla.
2. The part of fallopian tube closer to
(3) In between seminiferous tubules
ovary: leydig cells and some
(1) Ampula (2) Isthmus immunologically competent cells are
(3) Infundibulum (4) Fundus present.
(4) By the end of the second month of
3. Each testis has how many testicular pregnancy, the foetus develops
limbs and digits.
lobules:
(1) 100 (2) 150 (3) 250 (4) 750 9. Ovulation is:
(1) Releasing of secondary oocyte from ovary
4. The glans penis is covered by a loose (2) Releasing of primary oocyte from ovary
(3) Releasing of polar body
fold of skin is called:
(4) Releasing of graffian follicle
(1) Foreskin (2) Spermatic cord
10. For normal fertility, how many
(3) Gubernaculum (4) Fimbriae percentage of sperm must have normal
shape and size:
5. The fimbriae help in: (1) 50 % (2) 25 % (3) 40 % (4) 60 %
(1) Collection of the ovum after ovulation
11. In which duration of menstrual cycle
(2) Maintain the shape of ovary both LH & FSH attain a peak level:
(3) Provide the path to sperm during (1) In last week
(2) Middle of cycle
fertilization
(3) Initial days of cycle
(4) Release of ovum from ovary. (4) 4th day of cycle

6. Which statement is wrong ? 12. Which statement is not correct ?


(1) In the absence of fertilization, the
(1) Mammary lobes containing clusters
corpus luteum degenerates
of cells called alveoli (2) During pregnancy all events of
(2) Uterus is also called womb menstrual cycle stop
(3) The secretion of LH & FSH decreases
(3) The last part of the oviduct is called
gradually during the follicular phase
Ampulla
(4) The menstrual flow results due to
(4) Stroma of ovary divided into two breakdown of endometrial lining
zones - cortex and medulla
13. In which phase of menstrual cycle
7. Spermatogenesis start at puberty due to Graffian follicle transform as the corpus
significant increase in the secretion of: luteum.

(1) GnRH (2) Ertrogen (1) Luteal (2) Proliferation

(3) Oxytocin (4) Progesterone (3) Follicular (4) Growth

63
14. Which hormones is essential for 21. Which hormone level reaches peak
maintenance of the endometrium? during the luteal phase of menstrual
(1) FSH (2) LH cycle?
(1) Luteinising hormone
(3) Progesterone (4) Testosterone
(2) Progesterone
(3) FSH
15. Which of the following is an indicator of
(4) Estrogen
normal reproductive phase and extends
between menarche and menopause? 22. Placenta contains:
(1) Only chorionic villi
(1) Menstruation cycle (2) Estrous cycle
(2) Only uterine tissue
(3) Ovulation (4) Implantation (3) Chorionic villi + uterine tissue
(4) Trophoblast + chorionic villi
16. Fertilization takes place at:
23. The average duration of human
(1) Cervix
pregnancy is about nine months which is
(2) Ampullary region of fallopian tube called:
(3) Infundibulum region of fallopian tube (1) Gestation period (2) Parturition
(3) Lactation (4) Implantation
(4) Uterus
24. Parturition is induced by:
17. The embryo with 8 to 16 blastomeres is (1) A complex neuroendocrine mechanism
called: (2) A simple neuroendo crine mechanism
(1) Morula (2) Blastula (3) A neuro exocrine mechanism
(3) Gastrula (4) Foetus (4) A physio-chemical mechanism

25. Which hormon acts on the uterus during


18. During fertilization, a sperm comes in parturition ?
contact with which layer of the ovum. (1) Oxytocin (2) LH
(1) Jelly coat (3) Estrogen (4) Relaxin
(2) Zona pellucida 26. Which gland of female undergo
(3) Vitelline membrane differentiation during pregnancy ?
(4) Perivitelline space (1) Thyroid (2) Mammary
(3) Pituitary (4) Thymus
19. Function of placenta is:
27. Which is correct for colostrum ?
(1) Supply of O2 to embryo (1) It contains severel antibodies
(2) Removal of CO2 produced by the embryo (2) It produced during the last days of
lactation
(3) Produces several hormones
(3) It is a pheromone
(4) All of above (4) It is white in colour

20. The first sign of growing foetus may be 28. Placenta acts as an endocrine tissue and
produces several hormones like:
noticed by:
A.Human chorionic gonadotropin (hCG)
(1) Listening to the heart sound
B. Human placental lactogen (hPL)
carefully through the stethoscope C.Estrogens D.Progesterone
(2) Apperance of hair E. FSH F. LH
(3) Apperance of head (1) A, B, E & F (2) B only
(4) Apperance of eye lids (3) A, B & C (4) A, B, C & D

64
29. Which of the following group of 30. In mammals the female secondary
hormones are produced in women only sexual characters are developed mainly
during pregnancy ? by the hormone
(1) hCG, hPL relaxin (1) Relaxin
(2) Estrogen, progesterone, hCG (2) Estrogens
(3) Cortison, prolactin, thyroxine (3) Progesterone
(4) Prolactin, progesterone, hCG (4) Gonadotropins

ANSWER KEY
Que. 1 2 3 4 5 6 7 8 9 10 11 12 13 14 15 16 17 18 19 20 21 22 23 24 25
Ans. 2 3 3 1 1 3 1 1 1 4 2 3 1 3 1 2 1 2 4 1 2 3 1 1 1
Que. 26 27 28 29 30
Ans. 2 1 4 1 2

65
Exercise – III (Previous Year Questions)
(AIPMT 2007) 7. Secretions from which one of the
1. In the human female, menstruation can following are rich in fructose, calcium
be deferred by the administration of: and enzymes ?
(1) FSH only (1) Male accessory glands
(2) LH only (2) Liver
(3) Combination of FSH and LH (3) Pancreas
(4) Combination of estrogen and progesterone (4) Salivary glands
8. Which one of the following statements
(AIPMT 2008) about morula in humans is correct ?
2. In humans, at the end of the first (1) It has more cytoplasm and more
meiotic division, the male germ cells DNA than an uncleaved zygote
differentiate into the:- (2) It has almost equal quantity of
(1) Spermatids cytoplasm as an uncleaved zygote
(2) Spermatozonia but much more DNA
(3) Primary spermatocytes (3) It has far less cytoplasm as well as
(4) Secondary spermatocytes less DNA than in an uncleaved
zygote
(AIPMT 2009)
(4) It has more or less equal quantity of
3. Foetal ejection reflex in human female is
cytoplasm and DNA as in uncleaved
induced by:
zygote
(1) Release of oxytocin from pituitary
(2) Fully developed foetus and placenta 9. The second maturation division of the
(3) Differentiation of mammary glands mammalian ovum occurs:
(4) Pressure exerted by amniotic fluid (1) In the Graafian follicle following the
first maturation division
4. A change in the amount of yolk and its
(2) Shortly after ovulation before the
distribution in the egg will effect: ovum makes entry into the Fallopian
(1) Pattern of cleavage tube
(2) Number of blastomeres produced (3) Until after the ovum has been
(3) Fertilization penetrated by a sperm
(4) Forrmation of zygote (4) Until after nucleus of the sperm has
fused with that the ovum
(AIPMT 2010)
10. Which one of the following statements
5. The part of fallopian tube closest to the
about human sperm is correct ?
ovary is
(1) Acrosome serves no particular function
(1) Ampulla (2) Isthmus
(2) Acrosome has a conical pointed
(3) Infundibulum (4) Cervix
structure used for piercing and
6. Signals from fully developed foetus and penetrating the egg resulting in
placenta ultimately lead to parturition fertilization
which requires the release of: (3) The sperm lysins in the acrosome
(1) Estrogen from placenta dissolve the egg envelope facilitating
(2) Oxytocin from maternal pituitary fertilization
(3) Oxytocin from foetal pituitary (4) Acrosome serves as a sensory
(4) Relaxin from placenta structure leading the sperm towards
the ovum

66
11. Sertoli cells are found in: (1) (I) Perimetrium, (II) Myometrium, (III)
(1) Pancreas and secrete cholecystokinn Fallopian tube
(2) Ovaries and secrete progesterone
(2) (II) Endometrium, (III) Infundibulum,
(3) Adrenal cortex and secrete adrenaline
(4) Seminiferous tubules and provide (IV) Fimbriae
nutrition to germ cells (3) (III) Infundibulum, (IV) Fimbriae, (V)
12. Vasa efferentia are the ductules leading Cervix
from: (4) (IV) Oviducal funnel, (V) Uterus, (VI)
(1) Epididymis to urethra Cervix
(2) Testicular lobules to Rete testis
(3) Rete testis to epididymis 17. The testes in humans are situated
(4) Vas deferens to epididymis outside the abdominal cavity inside a
13. Seminal plasma in human males is rich pouch called scrotum. The purpose
in:
served is for
(1) Ribose and potassium
(2) Fructose and calcium (1) Providing a secondary sexual feature
(3) Glucose and calcium for exhibiting the male sex
(4) DNA and testosterone (2) Maintaining the scrotal temperature
14. The first movements of the foetus and lower than the internal body
appearance of hair on its head are temperature
usually observed during which month of
(3) Escaping any possible compression
pregnancy?
(1) Third month (2) Fourth month by the visceral organs
(3) Fifth month (4) Sixth month (4) Providing more space for the growth

15. In human female the blastocyst: of epididymis


(1) Forms placenta even before
18. If for some reason, the vasa efferentia in
implantation
(2) Gets implanted into uterus 3 days the human reproductive system get
after ovulation blocked, the gametes will not be
(3) Gets nutrition from uterine transported from:
endometrial secretion only after
(1) Vagina to uterus
implantation
(4) Gets implanted in endometrium by (2) Testes to epididymis
the trophoblasts cells (3) Epididymis to vas deferens
(4) Ovary to uterus
(AIPMT 2011)
16. The figure given below depicts a 19. What happens during fertilisation in
diagrammatic sectional view of the humans after many sperms reach close
female reproductive system of humans.
to the ovum ?
Which one set of three parts out of I -
IV have been correctly identified ? (1) Cells of corona radiata trap all the
(AIPMT 2011) sperms except one
(2) Only two sperms nearest the ovum
penetrate zona pellucida
(3) Secretions of acrosome helps one
sperm enter cytoplasm of ovum
through zona pellucida
(4) All sperm except the one nearest to
the ovum lose their tails

67
20. Given below is an incomplete table 24. Which one of the following statements
about certain hormones, their source is false in respect of viability of
glands and one major effect of each on mammalian sperm?
the body in humans. Identify the correct (1) Sperms must be connected in a
option for the three blanks A, B and C: - thick suspension
Gland Secretion Effect on Body (2) Sperm is viable for only up to 24 hours
A Oestrogen Maintenance of secondary (3) Survival of sperm depends on the pH
sexual characters of the medium and is more active in
Alpha Cells of is B alkaline medium
Raises blood sugar level
lets of Langerhans (4) Viability of sperm is determined by
Anterior C Over secretion its motality.
pituitary leads to gigantism
(AIPMT Mains 2012)
Options
A B C 25. The secretory phase in the human
(1) Ovary Insulin Vasopressin menstrual cycle is also called:
(2) Ovary Insulin Calcitonin (1) Follicular phase and lasts for about 13 days
(3) Ovary Glucagon Growth hormone (2) Luteal phase and lasts for about 6 days
(4) Placenta Glucagon Calcitonin (3) Follicular phase lasting for about 6 days
(4) Luteal phase and lasts for about 14 days
(AIPMT 2012)
(NEET 2013)
21. The Leydig cells as found in the human
body are the secretory source of: 26. Which one of the following is not the
(1) Glucagon (2) Androgens function of placenta ? It:
(3) Progesterone (4) Intestinal mucus (1) Secretes estrogen
(2) Facilitates removal of carbon dioxide
22. In a normal pregnant woman, the and waste material from embryo
amount of total gonadotropin activity (3) Secretes oxytocin during parturition
was assessed. The result expected was: (4) Facilitates supply of oxygen and
(1) High level of circulating HCG to nutrients to embryo
stimulate estrogen and progesterone
synthesis 27. Menstrual flow occurs due to lack of:
(2) High level of circulating FSH and LH (1) FSH
in the uterus to stimulate (2) Oxytocin
implantation of the embryo (3) Vasopressin
(3) High level of circulating HCG to (4) Progesterone
stimulate endometrial thickening
28. What is the correct sequence of sperm
(4) High level of FSH and LH in uterus to
formation?
stimulate endometrial thickening
(1) Spermatogonia, spermatocyte,
spermatozoa, spermatid
23. Signals for parturition originate from:
(2) Spermatogonia, spermatozoa,
(1) Fully developed foetus only
spermatocyte, spermatid
(2) Both placenta as well as fully
(3) Spermatogonia, spermatocyte,
developed foetus
spermatid, spermatozoa
(3) Oxytocin released from maternal
(4) spermatid, spermatocyte,
pituitary
spermatogonia, spermatozoa
(4) Placenta only

68
(AIPMT 2013) 35. Which of the following cells during
29. The main function of mammalian gametogenesis is normally diploid ?
corpus luteum is to produce: (1) Secondary spermatocyte
(1) Extrogen only (2) Primary polar body
(2) Progesterone (3) Spermatid
(3) Human chorionic gonadotropin (4) Spermatogonia
(4) Relaxin only
(Re-AIPMT 2015)
(AIPMT 2014) 36. Ectopic pregnancies are referred to as:
(1) Pregnancies terminated due to
30. The shared terminal duct of the hormonal imbalance
reproductive and urinary system in the (2) Pregnancies with genetic abnormality
human male is: (3) Implantation of embryo at site other
(1) Urethra than uterus
(2) Ureter (4) Implantation of defective embryo in
(3) Vas deferens the uterus
(4) Vasa efferentia
37. Which of the following events is not
associated with ovulation in human
31. Select the correct option describing
female?
gonadotropin activity in a normal
(1) LH surge
pregnant female:
(2) Decrease in estradiol
(1) High level of FSH and LH stimulates (3) Full development of Graafian follicle
the thickening of endometrium. (4) Release of secondary oocyte
(2) High level of FSH and LH facilitate
implantation of the embryo. 38. In human females, meiosis-II is not
(3) High level of hCG stimulates the completed until:
synthesis of estrogen and (1) Birth
(2) Puberty
progesterone.
(3) Fertilization
(4) High level of hCG stimulates the
(4) Uterine implantation
thickening of endometrium.
39. Which of the following layers in an antral
(AIPMT 2015)
follicle is acellular?
32. Hysterectomy is surgical removal of: (1) Zona pellucida (2) Granulosa
(1) Mammary glands (2) Uterus (3) Theca (4) Stroma
(3) Prostate glands (4) Vas-deference (NEET 2016)
40. Fertilization in humans is practically
33. Which of these is not an important
feasible only if:
component of initiation of parturition in
(1) The sperms are transported into
humans ? vagina just after the release of ovum
(1) Release of prolactin in fallopian tube
(2) Increase in estrogen and (2) The ovum and sperms are
progesterone ratio transported simultaneously to
(3) Synthesis of prostaglandins ampullary isthmic junction of the
(4) Release of oxytocin fallopian tube
(3) The ovum and sperms are transported
34. Capacitation refers to changes in the: simultaneously to ampullary - isthmic
(1) Sperm after fertilization junction of the cervix.
(2) Sperm before fertilization (4) The sperms are transported into
(3) Ovum before fertilization cervix within 48 hrs of release of
(4) Ovum after fertilization ovum in uterus.

69
41. Select the incorrect statement: 46. Identify the correct statement on
(1) FSH stimulates the sertoli cells 'inhibin':-
which help in spermiogenesis (1) Inhibits the secretion of LH, FSH and
Prolactin.
(2) LH triggers ovulation in ovary
(2) Is produced by granulose cells in
(3) LH and FSH decrease gradually ovary and inhibits the secretion of FSH.
during the follicular phase (3) Is produced by granulose cells in
(4) LH triggers secretion of androgens ovary and inhibits the secretion of LH.
from the Leydig cells (4) Is produced by nurse cells in testes
and inhibits the secretion of LH.
42. Which of the following is incorrect (NEET 2017)
regarding vasectomy ?
(1) Vasa deferentia is cut and tied 47. Capacitation occurs in:
(1) Rete testis
(2) Irreversible sterility (2) Epididymis
(3) No sperm occurs in seminal fluid (3) Vas deferens
(4) No sperm occurs in epididymis (4) Female Reproductive tract

43. Which of the following dipicts the correct 48. GnRH, a hypothalamic hormone, needed
in reproduction, acts on:
pathway of transport of sperms ?
(1) anterior pituitary gland and
(1) Rete testis → Vas deferens → stimulates secretion of LH and FSH.
Efferent ductules → Epididymis (2) posterior pituitary gland and
(2) Efferent ductules → Rete testis → stimulates secretion of oxytocin and FSH.
(3) posterior pituitary gland and
Vas deferens → Epididymis
stimulates secretion of LH and relaxin.
(3) Rete testis → Efferent ductules → (4) anterior pituitary gland and
Epididymis → Vas deferens stimulates secretion of LH and
(4) Rete testis → Epididymis → Efferent oxytocin.
(NEET 2018)
ductules → Vas deferens
49. Hormones secreted by the placenta to
maintain pregnancy are:
44. Match Column-I with Column-II and
(1) hCG, hPL, progestogens, prolactin
select the correct option using the (2) hCG, hPL, estrogens, relaxin, oxytocin
codes given below: (3) hCG, hPL, progestogens, estrogens
(4) hCG, progestogens, estrogens,
Column I Column II glucocorticoids
a Mons pubis i Embryo formation
50. The difference between spermiogenesis
b Antrum ii Sperm and spermiation is:
Female external (1) In spermiogenesis spermatids are
c Trophectoderm iii
genitalia formed, while in spermiation
d Nebenkern iv Graffian follicle spermatozoa are formed.
Codes: (2) In spermiogenesis spermatozoa are
formed, while in spermiation
a b c d
spermatids are formed.
(1) iii i iv ii (3) In spermiogenesis spermatozoa from
(2) i iv iii ii sertoli cells are released into the
(3) iii iv ii i cavity of seminiferous tubules, while
(4) iii iv i ii in spermiation spermatozon are
formed.
45. Several hormones like hCG, hPL, (4) In spermiogenesis spermatozoa are
formed, while in spermiation
estrogen, progesterone are produced by:
spermatozoa are released from
(1) Fallopian tube (2) Pituitary sertoli cells into the cavity of
(3) Ovary (4) Placenta seminiferous tubules.

70
51. Match the items given in Column I with (NEET 2020)
those in column II and select the correct
option given below: 55. Which of the following hormone levels
Column-I Column-II will cause release of ovum (ovulation)
Breakdown of from the graffian follicle ?
a. Proliferative Phase i. endometrial (1) High concentration of Progesterone
lining (2) Low concentration of LH
b. Secretory Phase ii. Follicular Phase (3) Low concentration of FSH
c. Menstruation iii. Luteal Phase (4) High concentration of Estrogen
a b c 56. Meiotic division of the secondary oocyte
(1) iii i ii
(2) iii ii i is completed:
(3) ii iii i (1) At the time of copulation
(4) i iii ii (2) After zygote formation
(NEET 2019) (3) At the time of fusion of a sperm with
52. Select the correct sequence for an ovum
transport of sperm cells in male (4) Prior to ovulation
reproductive system.
(1) Seminiferous tubules → Vasa 57. Match the following columns and select
efferetia Epididymis → Inguinal the correct option.
canal → Urethra
Column-I Column-II
(2) Testis → Epididymis → Vasa
(a) Placenta (i) Androgens
efferentia → Vas deferens →
Ejaculatory duct → Inguinal canal → (b) Zona pellucida (ii) HumanChoronic
Urethra → Urethral meatus Gonadotropin
(3) Testis → Epididymis → Vasa (hCG)
efferentia → Rete testis → Inguinal
canal → Urethra (c) Bulbo-urethral (iii) Layer of the
(4) Seminiferous tubules → Rete testis
glands ovum
→ Vasa efferentia → Epididymis →
Vas deferens Ejaculatory duct →
Urethra → Urethral meatus (d) Leydig cells (iv) Lubrication of
the Penis
53. Extrusion of second polar body from egg
nucleus occurs: (a) (b) (c) (d)
(1) before entry of sperm into ovum (1) (i) (iv) (iii) (iv)
(2) simultaneously with first cleavage (2) (iii) (ii) (iv) (i)
(3) after entry of sperm but before (3) (ii) (iii) (iv) (i)
fertilization
(4) (iv) (iii) (i) (ii)
(4) after fertilization
(NEET 2020 (Covid-19))
(NEET(UG) 2019 (Odisha)) 58. In human beings, at the end of 12 weeks
54. No new follicles develop in the luteal
(first trimester) of pregnancy, the
phase of the mensturual cycle because:
(1) Follicles do not remain in the ovary following is observed: -
after ovlation (1) Eyelids and eyelashes are formed
(2) FSH levels are high in the luteal (2) Most of the major organ systems are
phase formed
(3) LH levels are high in the luteal phase
(3) The head is covered with fine hair
(4) Both FSH and LH levels are low in
the luteal phase (4) Movement of the foetus

71
59. Select the correct option of haploid cells (NEET 2022)
from the following groups: – 64. At which stage of life the oogenesis
(1) Primary oocyte, Secondary oocyte, process is initiated?
Spermatid (1) Puberty
(2) Secondary spermatocyte, First polar (2) Embryonic development stage
(3) Birth
body, Ovum
(4) Adult
(3) Spermatogonia, Primary spermatocyte,
Spermatid 65. Which of the following statements are
(4) Primary spermatocyte, Secondary true for spermatogenesis but do not
spermatocyte, Second polar body
hold true for Oogenesis?
(a) It results in the formation of haploid
60. Match the following columns and select
gametes
the correct option: –
(b) Differentiation of gamete occurs
Column - I Column - II
after the completion of meiosis
Human chorionic
(a) Ovary (i) (c) Meiosis occurs continuously in a
Gonadotropin
mitotically dividing stem cell
Estrogen &
(b) Placenta (ii) population
Progesterone
(d) It is controlled by the Luteinising
(c) Corpus luteum (iii) Androgens
hormone (LH) and Follicle
(d) Leyding cells (iv) Progesterone Only
Stimulating Hormone (FSH) secreted
(1) (a)–(iv), (b)–(iii), (c)–(ii), (d)–(i)
by the anterior pituitary
(2) (a)–(i), (b)–(ii), (c)–(iii), (d)–(iv)
(3) (a)–(i), (b)–(iii), (c)–(ii), (d)–(iv) (e) It is initiated at puberty
(4) (a)–(ii), (b)–(i), (c)–(iv), (d)–(iii) Choose the most appropriate answer
from the options given below:
(NEET 2021) (1) (c) and (e) only
(2) (b) and (c) only
61. Receptors for sperm binding in (3) (b), (d) and (e) only
mammals are present on: (4) (b), (c) and (e) only
(1) Corona radiata
(2) Vitelline membrane 66. Given below are two statements :
(3) Perivitelline space Statement I :
(4) Zona pellucida The release of sperms into the
seminiferous tubules is called
62. Which of these is not an important spermiation.
component of initiation of parturition in Statement II : Spermiogenesis is the
humans? process of formation of sperms from
(1) Increase in estrogen and spermatogonia.
progesterone ratio In the light of the above statements,
choose the most appropriate answer
(2) Synthesis of prostaglandins
from the options given below :
(3) Release of Oxytocin
(1) Both Statement I and Statement II
(4) Release of Prolactin
are correct
(2) Both Statement I and Statement II
63. Which of the following secretes the
are incorrect
hormone, relaxin, during the later phase (3) Statement I is correct but
of pregnancy? Statement II is incorrect
(1) Graafian follicle (2) Corpus luteum (4) Statement I is incorrect but
(3) Foetus (4) Uterus Statement II is correct

72
(Re-NEET 2022) (NEET 2023)
67. Arrange the components of mammary 70. Given below are two statements:
gland. (from proximal to distal) Statement I: Vas deferens receives a duct
(a) Mammary duct from seminal vesicle and opens into
(b) Lactiferous duct urethra as the ejaculatory duct.
(c) Alveoli Statement II: The cavity of the cervix is called
(d) Mammary ampulla cervicle canal which along with vagina
(e) Mammary tubules forms birth canal.
Choose the most appropriate answer In the light of the above statements ,
from the options given below : choose the correct answer from the
(1) (c) → (a) → (d) → (e) → (b) options given below:
(2) (b) → (c) → (e) → (d) → (a) (1) Both Statement I and Statement II are
(3) (c) → (e) → (a) → (d) → (b) true.
(2) Both Statement I and Statement II
(4) (e) → (c) → (d) → (b) → (a)
are false.
68. How many secondary spermatocytes are (3) Statement I is correct but Statement
required to form 400 million spermatozoa ? II is false.
(1) 50 million (2) 100 million
(4) Statement I is incorrect but
(3) 200 million (4) 400 million
69. Given below are two statements : one is Statement II is true.
labelled as Assertion (A) and the other is
71. Which of the following statements are
labelled as Reason (R).
correct regarding female reproductive
Assertion (A) :
cycle.
During pregnancy the level of thyroxine
(A)In non -primate mammals cyclical
is increased in the maternal blood.
changes during reproduction are
Reason (R) :
called oestrus cycle.
Pregnancy is characterised by metabolic
(B)First menstrual cycle begins
changes in the mother.
at puberty and is called menopause.
In the light of the above statements,
(C)Lack of memstruation may be
choose the most appropriate answer
indicative of pregnancy.
from the options given below :
(D)Cyclic menstruation extends between
(1) Both (A) and (R) are correct and (R) is
menarche and menopause.
the correct explanation of (A)
(1) A and D only
(2) Both (A) and (R) are correct but (R) is
(2)A and B only
not the correct explanation of (A)
(3)A,B and C only
(3) (A) is correct but (R) is not correct
(4) A,C and D only
(4) (A) is not correct but (R) is corrects

ANSWER KEY
Que. 1 2 3 4 5 6 7 8 9 10 11 12 13 14 15 16 17 18 19 20 21 22 23 24 25
Ans. 4 4 2 1 3 2 1 2 3 3 4 3 2 3 4 3 2 2 3 3 2 1 2 2 4
Que. 26 27 28 29 30 31 32 33 34 35 36 37 38 39 40 41 42 43 44 45 46 47 48 49 50
Ans. 3 4 3 2 1 3 2 1 2 4 3 2 3 1 2 3 4 3 4 4 2 4 1 3 4
Que. 51 52 53 54 55 56 57 58 59 60 61 62 63 64 65 66 67 68 69 70 71
Ans. 3 4 3 4 4 3 3 2 2 4 4 4 2 2 4 3 3 3 1 1 4

73

You might also like